Sie sind auf Seite 1von 104

SmartCAT-10720

Verbal Ability and Reading comprehension


Directions for Questions 1 - 3: Read the below passage and answer the questions that
follow

Starting with the pioneering and still unsurpassed Virtual History, a collection of
scintillating essays edited by Niall Ferguson in 1997, an unceasing stream of books and
essays has appeared.

Andrew Roberts, Robert Cowley, Geoffrey Parker and many others have edited further
collections. The prolific Jeremy Black has, inevitably, weighed in with a short survey of
the genre. Military historians have produced hundreds of essays on what might have
happened had this or that general adopted different tactics in this or that battle.
Dominic Sand brook wrote a sequence of 40 counterfactual essays for the New
Statesman. Iain Dale and his collaborators at Bite back Publishing produced a string of
collections imagining what things might have been like had Michael Portillo, or any one
of a number of other politicians, become prime minister instead of the people who
actually did. The cascade of books and essays seems never-ending. But how do we
account for this trend?

Before the mid-1990s such speculations were few and far between. Occasional asides can
be found in the works of historians going back to the ancient Greeks, but it took the end
of Providentialist history, which viewed all events as part of the working-out of God‘s
purposes, and the advent of the Romantic view of the past as a succession of epochs, each
essentially different from the previous one, before writers began to speculate at greater
length on what might have been.

The two earliest extended essays in the genre were both French. Louis-Napoléon
Geoffroy‘s The Apocryphal Napoleon imagined what would have happened if the
emperor had conquered Russia in 1812 instead of being defeated at Moscow; he would,
in Geoffroy‘s alternative version, have gone on to conquer the known world eventually
being crowned by the pope with the hereditary title ‗The All-Powerful‘. Later in the 19th
century, Charles Renouvier coined the term ‗Uchronia‘ (in a novel of the same name) to
denote ―the rewriting of history not as it was, but as it could have been‖.

Both writers had axes to grind. Geoffroy was Napoleon‘s adopted son; he wrote his essay
in the mid-1830s, when Bonapartism was beginning to re-emerge as a political force.
Renouvier, for his part, was prompted by Napoleon III‘s close alliance with the church to
imagine a history of Europe based on the survival of a tolerant, multi-faith Roman
empire. As these fantasies suggested, wishful thinking, along with a clear political
purpose, has been a prime constituent of counterfactual history from the outset.

At the same time, however, this kind of writing has always coexisted with a view of
‗what-if?‘ history as an amusing

Entertainment – sometimes in the same book or article. In 1931 the first collection of
essays in the genre – If It Had Happened Otherwise, edited by Sir John Collings Squire –
presented two articles that adopted a procedure opposite to wishful thinking. GM
Trevelyan‘s piece imagined the grim fate that England would have met had Napoleon
won the battle of Waterloo, while Monsignor Ronald Knox‘s contribution depicted a
Soviet-style regime that he posited would have descended upon Britain had the General
Strike of 1926 been successful. The political thrust of such dystopian fantasies was
obvious enough, but they have always represented a minority strand in the literature.

The publication of those essays perhaps reflected the political uncertainties of the early
1930s. But they had no successor for nearly half a century until 1979, when Daniel
Snowman edited If I Had Been… Ten Historical Fantasies, in which 10 historians
showed how they could have done better than the personalities they discussed –for
example, by preventing American independence or avoiding the First World War. These
essays were a product, perhaps, of the sentiment spread by Margaret Thatcher and her
government – that Britain had taken a wrong turning in 1945, with the retreat from
empire and the foundation of the welfare state.

1. The author indulges in ―if-then essays and analysis of existing History‖ to reflect that
A) The particular genre is essentially a set of fantasies about a probable better world
People have a chronic disapproval of history because it seems nothing but fabrication
B)
of facts
People who disapprove of historical facts tend to create a counterfactual narrative in
C)
order to further their ideology.
Recorded history is primarily assumed as political fabrication or suppression of facts
D)
and is thus fiction and should entertain alternate explanations

Answer Key: C) People who disapprove of historical facts tend to create a counterfactual
narrative in order to further their ideology.
SOLUTION:

The correct choice is C

The ―better world‖ concept is irrelevant to the passage because there is no inferential
link. Similarly, absolute statements are dangerous in the verbal section: ―chronic
disapproval‖ must be eliminated. Option 4 seems viable but maybe a comment on the
passage but not what the author may presume, according to the passage. As per the author
the purpose in these attempts is political.

2. It can be inferred that writing and studying ‗alternate history‘ has not only interesting
literary attributes but is also relevant in

1. Developing counter-conceptions to streamlined pedagogy resulting in enhanced


research

2. Developing multi-faceted case studies in management research increasing efficiency


in scenario building and decision making

3. Creating programs or tools that would better speculate future economic and market
trends

4. Manufacturing concepts and social models in social Psychology with immaculate


precision
A) All of the above
B) 1 & 2
C) 2 only
D) 2 & 3

Answer Key: D) 2&3

SOLUTION:

The correct choice is D

Strategy management is a creative area, though measured and accountable. Alternate


hypothesis of existing situation helps create workshop modules, which fosters creative
solutions to different possibilities of cases. We can agree to ―streamlined‖, because
alternate hypothesis is the contrast to ―streamlined‖ but not pedagogy, which is
not supported by the passage. And, as in option 4, concepts can‘t be manufactured, they
are developed, and rarely with immaculate precision in social psychology.

3. ―The music contains the musician‖ – taking this comment as true, what would be the
probable style statement made by the passage?
A) Analysis of an abstract
B) Critique on a genre
C) Abstract of an genre
D) Appreciation of an abstract

Answer Key: B) Critique on a genre

SOLUTION:

The correct choice is B

Alternate hypothetical history, as treated in the passage, is a genre, much established,


though not a fashion in practice. The passage is a critique of this trend or genre. The other
options can be eliminated because the passage cannot be an abstract if it is a critique.

Directions for Questions 4 - 6: Answer the questions based on what is stated or implied
in the passage below.

FOR a decade or more, the Spanish economic galleon has been blessed with a following
wind and full sails. It has outrun the OECD average in nine of the past ten years and the
euro-area standard for all of the past dozen. A country that in 1994 had an unemployment
rate of almost one in five has provided work for lots of immigrants as well as many more
of its natives. Almost two-fifths of net new jobs in the euro zone since the creation of the
single currency have been Spanish ones. Only a few years ago, the thought of Spanish
fashion chains, banks and construction companies swashbuckling their way around the
globe in search of booty would have seemed preposterous. Now, as our special report
describes, they are doing precisely that. Yet in home waters at least, more difficult
conditions are ahead—and Spain's weaknesses are about to be exposed.

For some time two hazards have been visible. One is a giddying rise in house prices,
which have climbed by 180% in the past decade, more than doubling in real terms. The
market has so far been steadying— property-price inflation fell to 7.2% in the year to the
first quarter—but the recent collapse of a property company's share price shows that the
stock market, at least, is worried. No wonder, when the market is overvalued and
oversupplied and house building accounts for 7-10% of GDP, depending on your
measure.

The second is the country's current-account deficit, which in absolute terms trails only
that of the United States. At more than 9% of GDP, it mainly reflects Spanish business's
thirst for borrowing. Lending to companies has risen by 30% in the past year. The euro
zone's central bankers are fond of repeating that its members' current accounts are no
more meaningful than those of Tennessee or Texas, but Spain's deficit does tell you
something: how tilted towards domestic demand—including construction—the country's
economy has become.

The booms in building and borrowing have been helped along by Spain's membership of
the euro zone, which has made credit much cheaper for people and businesses. In the run-
up to the creation of the single currency, Spain benefited as its interest rates tumbled
towards German levels. Since the euro came into being, monetary conditions
have remained pretty loose. Spain's inflation rate has consistently exceeded the euro-area
average by a percentage point or more, making its real interest rates correspondingly
lower and giving an extra puff to an economy already going at a rate of knots.

Granted, there is more to the Spanish story than the cheapness of borrowing in Euros
rather than pesetas. Like Ireland, which has also had a vertiginous housing boom, Spain
has a high proportion of people of the age to buy a first home and start a family. The rise
in female employment has increased families' incomes and what they are willing to pay
for a home. Many foreigners as well as Spaniards have had both appetite and wherewithal
for a second home in the sun and by the sea. Still, cheap money has played an important
part.

Now, though, Spain may be about to see the other side of life in the euro zone: interest
rates are rising and the currency is climbing, just as the economy is set to slow down.
Even though economists think growth stayed strong in the first quarter (perhaps 4% in the
past year), it is likely to lose strength— maybe abruptly, if the housing market is unkind.

In some ways, Spain is well placed for this test. The government has run a budget surplus
for the past two years (even allowing for the economic cycle) and gross debt is only
around 40% of GDP, so fiscal policy can help out should the economy slow sharply. In
other ways, though, it is poorly prepared. One obvious means of rebalancing the
economy, devaluing the currency, is ruled out, so Spain must find another method of
bringing down its real exchange rate. It will have to look hard. Wages have been hitched
to the country's higher-than-average inflation rate. Productivity growth has been woeful
(even though, admittedly, to some extent this reflects high employment growth). The
result has been a 12% increase in unit labour costs, relative to the euro-area average,
since 2000.

Spain's ―dual‖ labour market is no model, despite its remarkable job-creation record. That
as many as a third of workers are on temporary contracts suggests some flexibility. But
there are so many such contracts precisely because employers find permanent workers
expensive to fire—and thus to hire. Recent reforms have done too little to close the gap in
costs between the two types of contract. In product markets, too, Spain should do more to
loosen its economy and let in more competition. In the past few years, Germans have
found that it is possible to win competitiveness in a currency union. But they have also
found that it can hurt. So might the Spanish.

4. Why does the author say that devaluing the currency is ruled out?
A) Spain‘s has a high inflation rate
B) Spanish business‘s have borrowed heavily and will suffer if devaluation occurs
While Spain‘s current account deficit is high, overall debt is moderate at 40% of
C)
GDP
Spain has taken membership of euro zone and has replaced its currency with the
D)
Euro

Answer Key: D) Spain has taken membership of euro zone and has replaced its currency
with the Euro

SOLUTION:

The correct choice is D

As mentioned Spain has adopted the Euro in place of its old currency the ‗Peseta‘. Since
Euro is a common currency, no individual country can go for devaluation.

5. Why, according to the author have labour costs gone up in Spain?


A) As many as a third of workers are on temporary contracts
B) Wages have gone up in tandem with inflation, which has been high.
C) Wages for permanent workers are high and those of contract workers low
D) Most workers are employed in the real estate sector which is doing badly
Answer Key: B) Wages have gone up in tandem with inflation, which has been high.

SOLUTION:

The correct choice is B

Look at the second largest paragraph. Wages are linked to higher than average inflation.
So as inflation rises, wages also rise. Option C is factually true but doesn‘t explain
why labour costs have gone up.

6. The author is most likely to agree with which of the following regarding Spain‘s future
Unless Spain does something about its current account deficit and the rise in housing
A)
prices, it will not be able to carry forward it‘s economic boom
B) Spain is bound to meet Germany‘s fate
C) Spain is well placed to face the test if things go wrong
There is more to Spanish story than the cheapness of borrowing in Euros than
D)
pesetas

Answer Key: A) Unless Spain does something about its current account deficit and the
rise in housing prices, it will not be able to carry forward it‘s economic boom

SOLUTION:

The correct choice is A

The last line of the 1st Para states that ―Spain‘s weaknesses are about to be exposed‖, and
the 1st line of the 2nd Para states ―For some time two hazards have been visible‖. These
two are: rise in house prices and current-account deficit. So obviously if Spain is to take
care of its weaknesses then it needs to do something about these ‗hazards‘.

Direction for the questions 7 – 12: Read the following passages and answer the following
questions.

A number of frauds have come to light. Many more may be expected. The usual statement that
scams are found out during difficult times is true but banal. The question is the extent of
the corruption and whether it is systemic. There has never been any systematic discussion of the
importance of corruption for the functioning of the capitalist economy. It is assumed that it is not
systemic and only occurs in particular countries and at particular times for special reasons. At the
same time, it is hard to avoid a different conclusion when looking at the evolution of finance
capital.

Corruption is inevitable at the interface between government and private enterprise. Within
capitalism, the civil service or bureaucracies are worse paid and more constrained than their
peers in the private sector. It is inevitable that private enterprise will find a way of using its
advantages to sway opinion within the governmental institutions. The right attempts to argue that
government is necessarily inefficient and/or corrupt as compared with the private sector, because
the latter is controlled by the market. In reality, it is the other way around. Even if we assume
that bribery only occurs on the margins, there are more powerful ways of ensuring a contract is
delivered. The fact that government officials and politicians move into highly paid positions
within powerful companies is not because such people are either knowledgeable or intelligent,
but, in part, because they have the right contacts. Furthermore, many civil servants/bureaucrats
and businessmen live within a similar ambience so that the result is a more subtleform of
influence, which can only be called corruption in a very broad sense. The fact that a number of
large companies have been accused of bribing third world governments is not an accident.

In addition, the present denouement has much to do with the nature of finance capital itself and
the way it has evolved. The so-called lack of transparency of the $596 trillion of over the counter
financial derivatives was itself an open invitation to fraud. Indeed one may conclude that many
of them were an inducement to fraud even if technically legal. Giving mortgages to people who
were known to be unable to pay back is one example. Another is the packaging off such
mortgage with other more salubrious loans. The credit default swaps (CDS) were the form of
financial derivatives Warren Buffet called weapons of financial mass destruction. The selling on
of what amounted to a form of financial betting, without the buyer often being aware of the real
risk, was morally dubious. One may perhaps ask whether the evolution of finance capital from
investment in imperialism and its wars of conquest to investment in itself was not progress. At
least more of the capitalist class itself lost out.

One cannot but conclude that finance capital, which itself is an aspect of the decline of the
system, having entered its own decline, has shot its bolt. In conditions of decline, it is to be
expected that there will be frantic efforts to raise profits by whatever means possible. Illegal
sectors such as drug production, smuggling and selling had already become absorbed into
financial capital. The attempts to prevent the laundering of money, particularly important since
9/11, cannot succeed against determined and wealthy proponents. While there is now a campaign
against the tax havens, where money can be wiped clean of its origins, it is unlikely to be able to
go very far, given the power of the wealthy and the integration of those havens in the normal
operation of big business. The inspection of business accounts by auditors paid by those who are
audited is, in itself, dubious but governments are loathe to change the system. Indeed, a
government inspection service would open itself up to the possibility of bribery or influence,
given the relatively poorer pay and long-term prospects of such government auditors.
The government/private sector interface involves the payment of taxes, the issuing of contracts
by government to private enterprise, the use of consultants etc. and it is inevitable that private
enterprise will play the dominant role. The examples of exorbitant charges to the military sector
in the United States are part of the interrelation between the public and private sectors. The huge
overruns in construction contracts have become normal. Only in particular instances are the
private companies held to account, as where there is a more left wing or incorruptible party in
power. The payment of taxes is more of a game for the seriously wealthy, who are able to
employ prestigious accounting firms and, where necessary, tax havens.

7. What is the purpose of the passage?


A) To highlight the extent of corruption in the economy today
B) To understand the role of government in corruption
C) Understanding the origin and the nature of corruption prevalent in the economy today
D) To highlight the faults in the system which corruption exploits

Answer Key: C) Understanding the origin and the nature of corruption prevalent in the
economy today

SOLUTION:

c)

The passage mainly talks about the source of corruption while identifying the way in which it is
related to the government and private organizations and other activities.

a) This is a part of the passage and the passage also deals with systematic corruption.

b) Other facets contributing to corruption are also discussed.

d) This is the purpose with which the passage is written. The faults in the system which
corruption exploits are not mentioned as the primary objective

8. What is an observation of private enterprises identified to be incorrect?


They are governed by markets and hence less efficient thus having a lion‘s share in
A)
corruption
Private enterprises are supposed to play a dominant role in corruption at the interface
B)
between the government and the private enterprises
C) Third world governments have been alleged to have taken bribes from large companies
D) The private sector is better in terms of compensation as compared to government sector
Answer Key: A) They are governed by markets and hence less efficient thus having a lion‘s
share in corruption

SOLUTION:

a)

The passage states ―The right attempts to argue that government is necessarily inefficient and/or
corrupt as compared with the private sector, because the latter is controlled by the market. In
reality, it is the other way around.‖

b) is stated in the last paragraph.

c) This is mentioned in the second paragraph.

d) is mentioned at the start of the second paragraph

9. What is an assumption made in the discussion of corruption?


Private companies are held to account in places where there is a more left wing party in
A)
power
Giving mortgages to people who were known to be unable to pay back is a practice
B)
followed
C) Corruption occurs at the intersection of government and private agencies
D) Corruption is specific to each occurrence in terms of the reason behind it

Answer Key: D) Corruption is specific to each occurrence in terms of the reason behind it

SOLUTION:

d)

This is stated in the start of the first paragraph to be an incorrect assumption.

a), b), c) are all facts stated in the passage in the last, third and second paragraph respectively.

10. Which of the following statements about finance capital are true according to the passage?
I. It is now associated with illegal activities like smuggling.

II. It shows a lack of transparency with regards to its derivatives.

III. It is facing a decline in the current global situation


A) II and III
B) Only II
C) I and III
D) All of the above

Answer Key: D) All of the above

SOLUTION:

d)

I is mentioned in the fourth paragraph.

II is mentioned in third paragraph.

III is mentioned in fourth paragraph

11. Why does the author believe that the sale of CDS was immoral?
The mortgage loans were made to people who were known to be financially incapable of
A)
repaying those loans
B) CDS were basically a form of gambling rather than financial investments
The financial risk involved in CDS was too high and ended up bankrupting financial
C)
corporations
D) CDS were sold to people who were not aware of the risk that they were taking on

Answer Key: D) CDS were sold to people who were not aware of the risk that they were taking
on

SOLUTION:

Solution: We are taking about CDS and not about sub-prime loans. So option 1 is out. The
objection that the author has that buyer was not aware of the real risk involved. Option 4

12. What is the problem identified by the author with respect to auditors?
A) Government auditing may give rise to more corruption as they are likely to be underpaid
B) Auditors help the wealthy avoid taxes through various methods including tax havens
The Auditors are paid by the very businesses that they audit, hence are amenable to
C)
influence
Corruption is endemic to any government private relationship including taxes and
D)
government contracts

Answer Key: C) The Auditors are paid by the very businesses that they audit, hence are
amenable to influence

SOLUTION:

Solution: The question refers to the problem with the current system of auditors, which is that
auditors are hired and paid by the very businesses that they audit. Government auditing is an
alternative which according to the author is also susceptible to bribery and influence. Hence
option 3 is better than option 1. Option 2 is wrong as it is accounting firms and not auditors that
help the wealthy avid taxes. Option 4 is outside the scope of the question. Option 3

Direction for the questions 13 – 18: Read the following passages and answer the following
questions.

Everyone knows that the baby-boomer generation is in the process of retiring, and that all those
ex-hippies and punks can expect to live longer than the Americans who retired before them. But
the financial challenge this poses is less well understood. Any lingering complacency ought to
be exploded by two papers in the latest Journal of Retirement. The first, from the Centre for
Retirement Research (CRR) at Boston College, estimates the proportion of 65-year-olds who
will be able to retire without a big hit to their disposable income. Pensioners do not usually need
as much money coming in as workers: for a start, they no longer need to save for retirement. The
CRR estimates that 65-85% of their previous income is a reasonable ―replacement rate‖,
depending on the type of household.

As well as private pensions, elderly Americans receive income from the federal government (in
the form of Social Security, the public pension) and many earn money from their accumulated
wealth, particularly by taking equity out of their houses. Even allowing for these sources of
income, the CRR estimates that 52% of Americans may not be able to maintain their standard of
living (which it defines as having an income that falls no more than 10% below the replacement
rate).

Unsurprisingly, the biggest problems face those with no private pension at all: 68% of these
Americans are expected to fall short. Those lucky enough to be covered by defined-benefit
plans—in which pensions are linked to a worker‘s salary—have the least difficulty: only 20% are
deemed at risk. Of those in defined-contribution (DC) plans—in which workers receive whatever
pension pot they have accumulated by retirement—53% probably will not reach the replacement
rate. The problem is that many people simply do not save enough in a DC pension. The
combined contributions of employers and employees average just 11.3% of salary. This will not
generate the same level of pension as a typical defined-benefit plan. The CRR found that the
average retirement assets of those aged 50-59 were just $110,000 in 2013, slightly lower than in
2010. This balance will improve over time, since DC plans are relatively new, but there is a long
way to go. If pensioners take an (inflation-adjusted) 4% a year from their pot, they will need
$250,000 just to generate an income of $10,000.

At least, you might think, Social Security will provide a basic income for the elderly. But the
second paper, by Sylvester Schieber, a former chairman of the Social Security Advisory Board,
points out that there are holes in the safety net. Retirement income is based on an average of
every worker‘s highest 35 years of earnings. But if a worker was not in employment for the full
35 years—because of sickness, an inability to find work or time spent caring for children or other
relatives—the missing years count as zeroes for the calculation. Mr. Schieber finds that people
whose total careers lasted just 10-19 years comprise 8% of pensioners, but just over half of the
poorest tenth of the elderly. Such workers could hardly have saved more for their retirement;
they had barely adequate income in the first place.

That could be a looming problem, given Social Security‘s finances. Payroll taxes on current
workers no longer exceed the benefits paid out, prompting the government to tap the surplus of
past years. On current projections, this will run out in 2034. After that, the cost of pensions could
still be met mainly by contributions from workers. But the politicians of the day may be forced to
consider benefit cuts. Mr. Schieber thinks the Social Security system is underfunded by around
25%. If an across-the-board benefit cut of that magnitude was applied, it would hit the poorest
workers hardest, since they have few alternative sources of income. At present, a 25% benefit cut
would reduce the income of the median pensioner by 12.3%. But the poorest tenth would lose
18.9% of their aggregate income.

So budding pension reformers will have to solve several problems simultaneously. First, they
need to stabilize Social Security while protecting (and indeed enhancing) the incomes of the
poorest. That should involve some combination of higher taxes, benefit cuts for the best-paid and
greater saving in the middle rungs of the income ladder—at the same time as ensuring that
people do not save too much too quickly, in case the impact on demand crashes the economy.
Good luck

13. Why is that retired people require less money than active workers (estimated 65-85% of
previous income)?
A) Retired people are living longer lives and hence face greater financial challenges
B) Retired have income generating assets including own homes and don't need to set aside
money for savings
Retired people tend to have lower expenses especially as children would have grown up and
C)
left home
D) Most retired people have Defined Benefit pension plans

Answer Key: B) Retired have income generating assets including own homes and don't need to
set aside money for savings

SOLUTION:

Solution: Option 2. The answer lies in the 2nd paragraph. Two reasons are given for the retired
needing lesser income. One not needing to save money for retirement and second, that they have
acquired assets which generate income, especially home equity. Option 2

14. Why, according to Mr. Schieber will Social Security fail to address retirement needs of the
poorest of the retired Americans?
Social security payments are based on average income for 35 years of earnings and for the
A)
poorest of those whose working life has been much lower
Social security payments cannot be covered by payroll taxes of working people and hence a
B)
cut in payments will have to be done
C) The savings made by the poor generates an income far lower than required
D) Most of the poor are not covered under defined benefit pension plans

Answer Key: A) Social security payments are based on average income for 35 years of
earnings and for the poorest of those whose working life has been much lower

SOLUTION:

Solution: Option 1. Schieber explains how social security payments are calculated. The
calculation is based on average of last 35 years of income. Now for people who did not earn
during a part of that period, the value of income will be taken as zero and the average will
fall. Option 1. Option 2 is correct to the extent that there is shortfall of funds in social security
but there is no guarantee that benefits will be cut, especially for the poorest. Option 3 and 4 talk
about savings and pension plans not social security

15. What is the primary difference between Defined Benefit and Defined Contribution pension
plans?
A) DB plans offer more income than DC plans
Under DB plans, 80% of people are adequately protected unlike DC plans where 53% are
B)
not adequately covered
C) DC plans require very high contributions to generate inflation adjusted income
Under DB plans, pension is linked to salary while under DC plans, pension is linked to the
D)
contribution made by person while working

Answer Key: D) Under DB plans, pension is linked to salary while under DC plans, pension is
linked to the contribution made by person while working

SOLUTION:

Solution: Option 4. The answer lies in the names itself. For DB plans the pension benefit is
defined (salary linked) while in DC plans, the contribution to be made is defined and the benefit
is based on the contribution made. Option 4

16. Which of the following has NOT been cited in the passage as a reason for the shortfall in
income for retired Americans?
A) Loopholes in social security plans, pension is based on 35-year average income
B) Savings and contributions to DC pension plans are too low to generate adequate income
C) Longer life expectancy and higher medical expenses
D) Gap between social security payouts and payroll taxes which finance the former

Answer Key: C) Longer life expectancy and higher medical expenses

SOLUTION:

Solution: Option 3. While longer life expectancy has been mentioned in the article, medical
expenses have not been cited. All other options have been cited in the passage. Option 3

17. Why does the author suggest that increasing savings be done moderately?
Increasing savings too fast will lead to a demand shortfall that will affect the economy
A)
negatively
B) There is strong likelihood that social security benefits might need to be cut
C) The contribution to DC plans needs to be higher to generate adequate income
D) More and more people should be brought under DB plans as opposed to DC plans
Answer Key: A) Increasing savings too fast will lead to a demand shortfall that will affect the
economy negatively

SOLUTION:

Solution: Option 1. Refer to the last line of the passage. While increasing savings will help
Americans post retirement, a drastic increase in savings will reduce demand (demand for
goods/services is based on consumer spending) hence affecting the economy. Option 1

18. Which of the following is NOT a source of Income for elderly Americans?
A) Social security payments
B) Private pension plans, either DB or DC
C) Income from acquired assets including home equity
D) Money remittances from children

Answer Key: D) Money remittances from children

SOLUTION:

Solution: Option 4. All options except option 4 have been mentioned in the passage. Option 4

Directions for Questions 19- 24: Read the below passage and answer the questions that follow

Apart from the peculiar tenets of individual thinkers, there is also in the world at large an
increasing inclination to stretch unduly the powers of society over the individual, both by the
force of opinion and even by that of legislation: and as the tendency of all the changes taking
place in the world is to strengthen society, and diminish the power of the individual, this
encroachment is not one of the evils which tend spontaneously to disappear, but, on the
contrary, to grow more and more formidable. The disposition of mankind, whether as rulers or
as fellow-citizens, to impose their own opinions and inclinations as a rule of conduct on others,
is so energetically supported by some of the best and by some of the worst feelings incident to
human nature, that it is hardly ever kept under restraint by anything but want of power; and as
the power is not declining, but growing, unless a strong barrier of moral conviction can be
raised against the mischief, we must expect, in the present circumstances of the world, to see it
increase.

It will be convenient for the argument, if, instead of at once entering upon the general thesis,
we confine ourselves in the first instance to a single branch of it, on which the principle here
stated is, if not fully, yet to a certain point, recognised by the current opinions. This one branch
is the Liberty of Thought: from which it is impossible to separate the cognate liberty of speaking
and of writing. Although these liberties, to some considerable amount, form part of the political
morality of all countries which profess religious toleration and free institutions, the grounds,
both philosophical and practical, on which they rest, are perhaps not so familiar to the general
mind, nor so thoroughly appreciated by many even of the leaders of opinion, as might have
been expected. Those grounds, when rightly understood, are of much wider application than to
only one division of the subject, and a thorough consideration of this part of the question will
be found the best introduction to the remainder.

The time, it is to be hoped, is gone by, when any defence would be necessary of the "liberty of
the press" as one of the securities against corrupt or tyrannical government. No argument, we
may suppose, can now be needed, against permitting a legislature or an executive, not
identified in interest with the people, to prescribe opinions to them, and determine what
doctrines or what arguments they shall be allowed to hear. This aspect of the question, besides,
has been so often and so triumphantly enforced by preceding writers, that it needs not be
specially insisted on in this place. Though the law of England, on the subject of the press, is as
servile to this day as it was in the time of the Tudors, there is little danger of its being actually
put in force against political discussion, except during some temporary panic, when fear of
insurrection drives ministers and judges from their propriety; and, speaking generally, it is not,
in constitutional countries, to be apprehended, that the government, whether completely
responsible to the people or not, will often attempt to control the expression of opinion, except
when in doing so it makes itself the organ of the general intolerance of the public. Let us
suppose, therefore, that the government is entirely at one with the people, and never thinks of
exerting any power of coercion unless in agreement with what it conceives to be their
voice. But I deny the right of the people to exercise such coercion, either by themselves or by
their government. The power itself is illegitimate.

The best government has no more title to it than the worst. It is as noxious, or more noxious,
when exerted in accordance with public opinion, than when in opposition to it. If all mankind
minus one, were of one opinion, and only one person were of the contrary opinion, mankind
would be no more justified in silencing that one person, than he, if he had the power, would be
justified in silencing mankind. Were an opinion a personal possession of no value except to the
owner; if to be obstructed in the enjoyment of it were simply a private injury, it would make
some difference whether the injury was inflicted only on a few persons or on many. But the
peculiar evil of silencing the expression of an opinion is that it is robbing the human race;
posterity as well as the existing generation; those who dissent from the opinion, still more than
those who hold it. If the opinion is right, they are deprived of the opportunity of exchanging
error for truth: if wrong, they lose, what is almost as great a benefit, the clearer perception and
livelier impression of truth, produced by its collision with error.

19. In the context of the passage, the author would agree with the statement:
the protection of democratic institutions is a must in order to maintain individual liberty in
A)
society
B) individual opinions, with no impact on the larger world, can be discarded
individual opinions, at all costs, need to be protected as they can act as the pathway to
C)
truth
D) the role of the press, though sacrosanct, is controlled by law at all times

Answer Key: C) individual opinions, at all costs, need to be protected as they can act as the
pathway to truth

SOLUTION:

(3)

Refer to the following lines: If the opinion is right, they are deprived of the opportunity of
exchanging error for truth: if wrong, they lose, what is almost as great a benefit, the clearer
perception and livelier impression of truth, produced by its collision with error.

These lines are in perfect resonance with option (3). The central theme of the passage revolves
around individual liberty and that is the central concern of the author of the passage.

Option (1) is not mentioned in the passage.

Option (2) goes against the passage.

Option (4) is contradictory in nature. In one part, it states that the press is sacred and in the
second part of the option, it counters the same sentiment

20. The author gives primacy to which out of the following?


A) the rights of the individual
B) the rights of the state
C) the rights of the press
D) both options (1) and (3)

Answer Key: A) the rights of the individual


SOLUTION:

(1)

The important thing here is to understand the meaning of the word ‘primacy’. It means ‘the
state of being first in importance’. The author regards the rights of the individual as the most
important. The rights of the press are important but they are not primary. This is why the
answer is option (1) and not option (4)

21. According to the author of the passage, the liberty of the press does not need to be
defended any longer because:
the need of the press enjoying liberty has been discussed in detail and it has been
A)
successfully enforced by the writers and thinkers of the time
B) the danger of the law being used against the press is non-existent in the present day world
No argument is needed for permitting the legislature or an executive to prescribe opinions
C)
to the press
D) both (1) and (2)

Answer Key: D) both (1) and (2)

SOLUTION:

(4)

Refer to the highlighted portions: The time, it is to be hoped, is gone by, when any defence
would be necessary of the "liberty of the press" as one of the securities against corrupt or
tyrannical government. No argument, we may suppose, can now be needed, against permitting
a legislature or an executive, not identified in interest with the people, to prescribe opinions to
them, and determine what doctrines or what arguments they shall be allowed to hear. This
aspect of the question, besides, has been so often and so triumphantly enforced by preceding
writers, that it needs not be specially insisted on in this place. Though the law of England, on
the subject of the press, is as servile to this day as it was in the time of the Tudors, there is little
danger of its being actually put in force against political discussion, except during some
temporary panic, when fear of insurrection drives ministers and judges from their propriety;
and, speaking generally, it is not, in constitutional countries, to be apprehended, that the
government, whether completely responsible to the people or not, will often attempt to control
the expression of opinion, except when in doing so it makes itself the organ of the general
intolerance of the public.
The two portions in bold highlight the reasons why the press no longer needs to be worried.
Option (3) simply picks a line from paragraph. This line simply re-states that the press is free but
it does not provide reasons for the same

22. According to the author of the passage, if the opinion of the government is in agreement
with the voice of the people:
then the government is justified to use coercion, though it should place some checks and
A)
balances
B) even then the government is not justified for it to use coercion
C) then the government should identify individual opinions which are discordant in nature
then the government should make sure that no individual opinions are there that go
D)
against it

Answer Key: B) even then the government is not justified for it to use coercion

SOLUTION:

(2)

Refer to the lines: Let us suppose, therefore, that the government is entirely at one with the
people, and never thinks of exerting any power of coercion unless in agreement with what it
conceives to be their voice. But I deny the right of the people to exercise such coercion, either by
themselves or by their government. The power itself is illegitimate.

These clearly point to option (2) as the correct answer.

Option (1) goes against these lines and options (3) and (4) are not mentioned in the passage

23. To what does the author refer to by the usage of the word ―the mischief‖?
A) The virtues and the vices inherent to human nature
B) The inherent predilection of humans to force upon others their own views
C) The inherent tendency of man to hunger unrestrainedly after power
D) The inherent tendency of mankind to reduce the power of the individual

Answer Key: B) The inherent predilection of humans to force upon others their own views

SOLUTION:
2.

Refer to the first paragraph‘s ―The disposition of mankind, whether as rulers or as fellow-
citizens, to impose their own opinions and inclinations as a rule of conduct on others‖. The
author talks about how this tendency is growing

24. ―The peculiar evil of silencing the expression of an opinion is that it is robbing the human
race‖

Why does the author call it the peculiar evil?

It is peculiar because it robs not only the existing generation but also the ones to come

It is peculiar because it harms the one silencing the expression of opinion more than it does
the one expressing it

It is peculiar because if the opinion has merit then it is lost to an error of judgment

It is peculiar because if the opinion lacks credibility then the opportunity to strengthen the
opposing argument is lost
A) 1 and II
B) II and III
C) III and IV
D) All

Answer Key: C) III and IV

SOLUTION:

3.

Refer to the last part of the last paragraph…it clearly explains the peculiar evil of silencing the
right to freedom of expressing opinion…‖If the opinion is right, they are deprived of the
opportunity of exchanging error for truth: if wrong, they lose, what is almost as great a benefit,
the clearer perception and livelier impression of truth, produced by its collision with error.‖

25. In the below question, there are five sentences/paragraphs. The sentence/
paragraph labelled 1 to 5 need to be arranged in the logical order to form a coherent
paragraph/passage. From the given options, choose the most appropriate option.

1. Another three planes were still in the area trying to help solve the nearly 2-week-
old aviation mystery, and another was on the way to look for two large objects a
satellite detected floating off the southwest coast of Australia about halfway to the
desolate islands of the Antarctic.

2. Two more Orions and an ultra-long-range Bombardier Global Express were still
scouring the area 2,300 kilometers (1,400 miles) from Western Australia, according to
the Australian Maritime Safety Authority.

3. A search on Thursday with four planes in cloud and rain found nothing, and so far
efforts on Friday were the same, with a Royal Australian Air Force P3 Orion plane
flying back to Australia.

4. The area in the southern Indian Ocean is so remote is takes aircraft four hours to fly
there and four hours back, and leaves them only about two hours to search

5. The satellite discovery raised new hope of finding the vanished jet and sent another
emotional jolt to the families of the 239 people aboard.

Answer Key: 14532

SOLUTION:

14532

Although A as a starter seems improbable we must remember this paragraph is a part of a


larger passage. ―Off the southwest coast of Australia‖ refers to the Indian Ocean which is
linked to 4. next 53 are linked be the idea of raising hopes and then finding nothing.
2 comes after 3 as it talks about two more.

26. In the below question, there are five sentences/paragraphs. The sentence/
paragraph labelled 1 to 5 need to be arranged in the logical order to form a coherent
paragraph/passage. From the given options, choose the most appropriate option.

1. Nadella, who was appointed CEO of the $78-billion company in February, talked
about the enormous opportunities in cloud computing.

2. "It is amazing though, to think of the advancements in technology over the past few
years and the opportunity for developers in India today."

3. "Having grown up in India, the idea that I would have the opportunity to talk to all
of you as CEO of Microsoft was beyond my wildest dreams. Admittedly, my interests
at that time were a bit more focused on cricket than on technology," Nadella said, and
went on to add,

4. "We are at a pivotal time in our industry - living in and developing for a mobile-
first, cloud-first world," he said

5. He believes India is a cloud-first and mobile-first country, with cloud emerging as a


potential game-changer to deliver applications on smart devices at a fraction of the
cost of traditional computing.

Answer Key: 32154

SOLUTION:

32154

3 acts as an introduction of who Nadella is. Next comes 2 where we he talks about
advancements in India, which is broad based about technology opportunities in general.
Next is 1 which starts off the discussion on cloud-computing. 5 explains
what Nadella thinks about India. 4 supports 5.

27. In the below question, there are five sentences/paragraphs. The sentence/
paragraph labelled 1 to 5 need to be arranged in the logical order to form a coherent
paragraph/passage. From the given options, choose the most appropriate option.

1. According to the findings of a study by Gaadi.com, Maruti 800 enjoys a high


demand in the used car segment with more and more Chennai consumers buying and
selling this classic model.

2. Amongst the cars from Maruti, Maruti 800 emerged as a clear winner followed by
Swift, Alto and Wagon R. Amongst the other hatchbacks, Indica led the way followed
closely Hyundai's Santro Xing and i20.

3. Namma Chennai loves its old world charm. Nothing else can explain its continuing
love affair with the Maruti 800 in the city's car market where this trusted car continues
to enjoy top billing.

4.. Conducted across eight cities, the study looked at the supply trends of used
cars and the corresponding demand for the same, highlighting the buying trends of the
used car consumers

5. Discussions in Chennai revealed that Maruti and Tata from the hatchback segment
topped the charts as the most preferred used car brands with equally high demand and
supply.

Answer Key: 31452

SOLUTION:

31452

3 introduces Chennai‘s love for Maruti cars. 1 starts the talk about the study conducted
and 4 talks about where the study was conducted. 5 talks of preferences in Chennai
for Maruti and Tata and 2 follows with details of car models from the above brands.

28. In the below question, there are four sentences/paragraphs. The sentence/
paragraph labelled 1 to 4 need to be arranged in the logical order to form a coherent
paragraph/passage. From the given options, choose the most appropriate option.

1. This coolness arises partly from opponents who have the law on their side and
partly from the incredulity of men.

2. The best way to fully appreciate the scope of challenge we face in shifting to a
Clean Energy System is to read Machiavelli‘s The Prince.

3. The innovator has for enemies all those who have done well under the old
conditions and lukewarm defenders in those who may do well under the new.

4. ‗It ought to be remembered that there is nothing more perilous than to take a lead in
introducing a new order of things.‘
Answer Key: 2431

SOLUTION:

2431

. 2-4 is a strong link because the quoted statement follows from what he has read. The 3-1
link is brought about the coolness of those who oppose the innovator.

29. Below question has a paragraph from which the last sentence has been deleted. From
the given options, choose the sentence that completes the paragraph in the most
appropriate way.

The truth is that India's economy tends to rise or fall with the global economy, not
with the party in power. For virtually every five-year period since 1980, Indian GDP
has grown at a rate about 1.5 percentage points faster than the emerging
world average. This somewhat faster growth has mainly come about because India is
a low-income economy and it is easier to grow fast from a low base. Over the course
of the last three governments, India's average GDP growth rate has ranked somewhere
between 40th and 50th place out of the 150 nations in the emerging world, whether
Congress or BJP was in charge. The last BJP-led government took power in 1999 and
presided over a five-year period of rather standard Indian performance. India posted
average annual GDP growth of 5.8% — ranking 50th in the emerging world — with
inflation of 3.9% — ranking 70th in the emerging world. This record represented little
change from the previous 20 years. _____________________________________

1. So it is hard to see why it should provoke much criticism, or bragging

2. So it is clear why the BJP government is still basking in the glory of the yesteryears

3. So it is clear that the Indian public are impressed with the BJP government

4. So it is clear that whichever government you choose, there will hardly be any
further growth

Answer Key: 1

SOLUTION:

1
The first sentence is actually the conclusion of the paragraph which gives us data on the
growth of GDP ―whether Congress or BJP was in charge‖. Hence neither government can
claim to be responsible for the growth. So neither criticism nor accolade can be laid at
either of the governments‘ doors. Option D tries to predict that the future will also follow
the same trend, irrespective of government, which is far fetched.

30. Below question has a paragraph from which the last sentence has been deleted. From
the given options, choose the sentence that completes the paragraph in the most
appropriate way.

In Ukraine, the West is reaping the whirlwind it has sowed in Russia in the past two
decades. Flush with its triumph in the Cold War the West treated Russia as a fallen
enemy who would never rise again and whose interests‘ and sensitivities could be
safely ignored. They cheated Mikhail Gorbachev when they solemnly promised him
not to move NATO borders to the East if he agreed to the unification of Germany.
________________________________

1. Putin could not allow Ukraine to slip out of his control because it is the ‗birthplace
of Russian civilization‘

2. Putin had a far stronger geo-political compulsion to interfere, the all- to-real
prospect of Ukraine joining the NATO.

3. Along with pursuing military encirclement of Russia and the line of deceptions, the
West has sought to disrupt Russia‘s efforts for the economic re-integration of ex-
Soviet states.

4. The stakes for Putin are very high. If Ukraine slides into his control his popularity
graph will shoot up but if not there will be a surge of discontent on the home front.

Answer Key: 3

SOLUTION:

The entire paragraph focuses on the role played by the West. Thus 3 is a natural
continuation. The Putin thread line takes the argument on a different plane altogether
31. Complete the following paragraph with a suitable sentence from among the options
given

According to her memoirs, her most vivid childhood memories were not of London
but of Cornwall, where the family spent every summer. These family holidays and
impressions of the landscape informed the fiction she wrote in later years. The sudden
loss of her parents led to the first of her several nervous breakdowns; she was even
briefly institutionalized. Although she was able to take courses of study, some even at
degree level, throughout her life, Woolf was plagued by periodic mood swings and
associated illnesses. _________

1. Unlike the limited educational resources available to the girls in her family, her
brothers were sent on to Cambridge, a difference she would resent for the rest of her
life.

2. Though such chronic instability often affected her social life, her literary
productivity continued unabated, with few breaks.

3. She married writer Leonard Woolf, and despite their low material status, the couple
shared a close bond throughout their marriage.

4. Woolf is considered a major innovator in the English language.

Answer Key: 2

SOLUTION:

The passage talks about writer Virginia Woolf, painting a picture of her childhood
followed by her personal losses and the toll they took on her health, including progressive
mental instabilities. Option B offers a befitting conclusion, as it continues on the note of
her instability and its ill effects, and how despite severe health problems, her
writing/literary accomplishments sustained and grew.

Option A opens up a completely different/fresh subject and does nothing to conclude the
passage: it talks about the gender discrimination shown in her family that left her with a
lifelong grudge. Option C talks about whom she married and how their marriage was
happy: this could be a somewhat irrelevant but happy conclusion, but it is not as good as
option B which rounds off the main themes covered in the passage. Option D is not
appropriate; it sounds more like an opening sentence for a discussion of Woolf‘s literary
abilities and accomplishments, and does not touch upon the theme of the passage at all.

32. The following questions has a paragraph from which the last sentence has been
deleted. From the given options, choose the sentence that completes the paragraph in
the most appropriate way.

By propounding ―Darwinism,‖ even scientists and science writers perpetuate an


impression that evolution is about unique man, unique book, and unique ―theory.‖
The ninth-century Buddhist master Lin Chi said, ―If you meet the Buddha on the road,
kill him.‖ The point is that making a master teacher into a sacred fetish misses the
essence of his teaching. So let us now kill Darwin. That all life is related by common
ancestry, and that populations change form over time, are the broad strokes and fine
brushwork of evolution. But Darwin was late to the party.
________________________________

1. All Darwin perceived was that selection must work in nature, too.

2. Scientists often attribute the success of a phenomenon to an individual, thus


mitigating the magnitude of that particular discovery or innovation.

3. That‘s why Darwin must go.

4. His grandfather, and others, believed new species evolved.

Answer Key: 4

SOLUTION:

The paragraph talks about how a philosophy is attributed to an individual. It further gives
the author‘s point of view that by indulging in such a practice often the essence of the
teaching propounded by that ‗larger than life‘ figure is missed. Option (4) aptly carries
forward the theme of the paragraph by giving a suitable example that ‗evolution‘ existed
even before Darwin . Note the last line ‗Darwin was late to the party‘.

. 33. In the question given below, 4 sentences are given, labelled 1, 2, 3, 4, and 5; of
these, 4 statements need to be arranged in the logical order to form a coherent
paragraph. From the given option, choose the option that does not fit the sequence.

1. Property prices have cooled down significantly in the last one year.

2. So has this led to a lower rental bill for tenants?

3. Sukhinder, a broker in the NCR region says that once the buildings in the
construction phase hit the market, a reverse swing is predictable.

4. Many existing homes also remain unavailable to tenants as procedures involved in


sewing up a contract get stuck in bureaucratic quagmires.

5. Common floor.com, a real estate portal show that rents are up by over 10% in
Delhi and Mumbai.

Answer Key: 4

SOLUTION:

Rentals is focus, 1 and 2 and 5 show that cooling property prices have not resulted in
lower rentals, but 4 suggests that as additional houses come into the market, the trend will
reverse. The hassles in getting a rent contract made is irrelevant. (nothing about this
explains the changes in last 1 year)

34. Four sentences below are labeled as 1, 2, 3 and 4. Of these, three sentences, when
arranged in a logical order, form a coherent paragraph. From the given options,
choose the option that contains the sentence that does not fit the sequence.

1. As one agent puts it, ―Anyone can sell a Patterson or a Rowling‖.

2. They are a breed unto their own, having come of age only over the last decade or
so, but on their way to becoming indispensible to some of the big players.

3. Essentially acting as agents for agents, sub-agents provide a measure of how an


agent‘s books might work in various markets.

4. 2013 was a good year, given the sales figures recorded for digitalized copies of
many of the classics we‘ve all grown up reading.
Answer Key: 1

SOLUTION:

All sentences except 4 talk about the growing breed of sub-agents in the publishing
industry. Sentence 1 contains two author names, which if recognized will give away that
the context is book publishing/selling. Sentence 2 talks about sub-agents being a newly
formed breed whose popularity is on the rise. Sentence 3 defines ‗sub-agents‘. Sentence
4 is the odd one out because it only talks about a year that saw good sales figures for e-
books (of one kind of books). Although broadly, the subject in 4 is still publishing/books,
it is not touching upon the topic of agents or sub-agents, which features directly in all the
other three sentences.
Data Interpretation and Logical Reasoning

Directions for questions 1 to 4: Given below are the constituents of three petroleum
samples (in percentages).

1. If the three samples are mixed in a way such that Petroleum Jelly accounts for 5% of
the mixture, what can be the maximum percentage of LPG?
A) 11%
B) 10.66%
C) 10.5%
D) 10%

Answer Key: B) 10.66%

SOLUTION:

Choice B
If Petroleum Jelly accounts for 5% in the mixture, Sample 2 could be put in any quantity
since it has 5% Petroleum Jelly. Sample 1 and 3 have 6% and 3% Petroleum Jelly. Hence
they should be mixed in the ratio 2 : 1 to get 5% Petroleum Jelly. Since Sample 2 has
minimum percentage of LPG among the three samples, the mixture will have a maximum
percentage of LPG when Sample 2 is the least i.e. is not present in this case. We
should mix only Sample 1 and Sample 3 in the ratio 2 : 1. The % of LPG in this case will
be. 2 ×11+103=10.66%

2. Any two of the three samples are taken and mixed in a proportion such that Diesel is
more than 27% and LPG is more than 9% in the mixture. One sample that necessarily
has to be used is
A) Sample 1
B) Sample 2
C) Sample 3
D) Any two of the three can be used

Answer Key: A) Sample 1

SOLUTION:

Choice A

Since both Sample 1 and Sample 2 have percentage of Diesel more than the required
27%, they could be mixed in any proportion and the mixture would have Diesel more
than 27%. Since LPG in Sample 1 is 11% and that in Sample 2 is 7%, the ratio of mixing
Sample 1 and Sample 2 should be more than 1 : 1, though not needed to actually be found
out so that LPG accounts for more than 9%. Exactly same situation exists with Sample 1
and Sample 3 but with Diesel and LPG revering roles. Both of them have LPG more than
9% and hence any ratio of mixing will do to maintain LPG levels more than 9%. So one
can mix in a way that percentage of diesel is more than 27% (Since Sample 1 has 28%
diesel and Sample 3 has 25% diesel, any ratio more than 2 : 1 will do)

However when Sample 2 and Sample 3 are mixed … Since Sample 2 has 31% diesel and
Sample 3 has 25% diesel, so that percentage of diesel in mixture is, say, 27%, the ratio of
mixing has to be greater than 1 : 2. To keep percentage of LPG more than 9%, the ratio of
mixing should be less than 1 : 2 (less of Sample 2 and more of Sample 3). Thus, both
diesel more than 27% and LPG more than 9% cannot be achieved simultaneously.
3. The three samples are kept in three test-tubes: T1, T2 and T 3, but it is not known
which sample is in which test-tube. Ten parts of T1 is mixed with two parts of T2 and
eight parts of T3. Petrol now accounts for 14.8% of the mixture. The Samples in the
three test-tubes in order are:
A) Sample 3, Sample 2, Sample 1
B) Sample 1, Sample 3, Sample 2
C) Sample 3, Sample 1, Sample 2
D) Sample 2, Sample 3, Sample 1

Answer Key: C) Sample 3, Sample 1, Sample 2

SOLUTION:

Choice C

Sample 1, 2, and 3 have 12%, 14% and 16% of Petrol respectively. To get 14.8% of
Petrol, Sample 3 should be used maximum since only then would it counter the effect of
Sample 1 and 2 to decrease Petrol content.

Checking option c),. (16% × 10 + 12% × 2 + 14% × 8)20=14.8%

Thus, T-1, T-2 and T-3 in order are Sample 3, Sample1, Sample 2.

4. If the three samples are mixed in a way such that petrol accounts for 14% then which
of the following statement is necessarily true?
A) Proportion of sample 3 should be double of sample1
B) Proportion of sample 2 should be more than sample 1
C) Proportion of sample 2 is dependent on proportion of both sample 1 and 3.
D) Proportion of sample 1 and 3 should be same

Answer Key: D) Proportion of sample 1 and 3 should be same

SOLUTION:
Choice D

We want percentage of petrol to be 14% and sample 2 already contains 14% of petrol. So
the mixture is not dependent on the proportion of sample 2. Now sample 1 and 3 contains
12% and 16% of petrol respectively. For overall percentage of

Petrol to be 14%, by method of allegation we can say that proportion of sample 1 and 3
should be mixed in the ratio of 1 : 1

Directions for questions 5 to 8: A building of P × R (units)2 floor area has Q floors. It


has P × R cubical rooms per floor. A room is given its number (x, y, z) where x, y, z is
the cubicle number along X, Y and Z axis respectively.

For example: If the grey shaded cubicle is given a room number of (12, 2, 1) it is the 12
room towards X-axis from ‗O‘ the entrance of the building, and located on the 2nd floor
and 1st room from Z-axis.
The black shaded cubicle is the cubicle on the extreme diagonal end and has its room
number as (P, Q, R)

5. If a cubicle which has only odd numbers in its room number gets even number of
toffees. A cubicle which has only even numbers in its room number will get odd
number of toffees. All the other cubicles will not get any toffees. Which of the
following are true?

I. If P is odd, Q is even and R is odd, the rooms which get even number of toffees will
be more than those which get odd number of toffees.

II. If P, Q and R are even, the rooms which get even number of toffees are more than
those which get odd number of toffees.

III. If P,Q and R are all odd, the number of rooms which get odd number of toffees
will be more than those which get even number of toffees.
A) I only
B) II only
C) III only
D) I and III only

Answer Key: A) I only

SOLUTION:

Choice A

I. Assuming P = 3, Q = 2 and R = 3: The cubicles that get even number of toffees are (1,
1, 1), (1, 1, 3), (3, 1, 1) and (3, 1, 3) i.e. 4. The cubicle that getodd number of toffeesis (2,
2, 2) i.e. only 1 cubicle in the grid. Thus, this statement is true. Now, looking at the
option choices, one just needs to check for statement III.

III. Assuming P = Q = R = 3 : The cubicles that get even number of toffees are (1, 1, 1),
(1, 1, 3), (1, 3, 1), (1, 3, 3), (3, 1, 1) and (3, 1, 3) i.e. 6 even numbers. The cubicles
that getodd number of tofeesis (2, 2, 2) i.e. only 1 cubicle is present in the grid. Thus, this
statement is not true.

Theoretically, if P is odd, Q is even and R is odd, then the number of cells (x, y, z) such

that each of x, y and z is odd will be and the number of cells (x, y,

z) such that each of x, y and z is even will be , which is obviously less


than the previous expression. Similar expressions can be found depending on P, Q and R
being even or odd.

6. Type the answer in the given text box:


If (P, Q, R) is (8, 8, 8) and in each cubicle bearing a room number (x, y, z), x+ y + z
toffees are given. What is the total number of toffees given to all the cubicles?

Answer Key: 6912

SOLUTION:

6912

Rather than consider the room number in a cubicle, say (2, 3, 1) as 6, one should consider
the number as 2 + 3 + 1 itself. It will help us add the numbers easily.

The number of cubicles with room number (1, y, z) will be 8 × 8 i.e. 64 cubicles.
Similarly the number of cubicles with room numbers (2, y, z) will be 64. And same for
(3, y, z), (4, y, z),….(8, y, z). Not just with the x co-ordinate, the same will be true i.e.
there will be 64 cubicles with room numbers (x, 1, z) or (x, 2, z)… (x, y, 1), (x, y, 2) …

Thus, when all the number of toffees are added each of 1, 2and 3 will be added 64 times
with they being the x co-ordinate, 64 times with they being the y co-ordinate and 64
times as z co-ordinates. Thus, each of 1, 2, 3 and 4 will be added 192 times. The required
sum = 192 × (1 + 2 + 3 + 4 +…+ 8) = 6912

7. If (P, Q, R) = (7, 7, 7) and in all the cubicles having three co-ordinates of the room
number equal, 2 toffees is placed. ‗A‘ refers to the total number of toffees placed in all
the cubicles.

If (P, Q, R) = (9, 10, 11) every cubicle that satisfies 6< x < 8, 5 < y < 9, 5 < z < 10 is
given one biscuit. If ‗B‘ refers to the total number of biscuits given to all the cubicles,
then which of the following is true about ‗A‘ and ‗B‘?
A) A > B
B) A < B
C) A = B
D) Cannot be determined.

Answer Key: A) A > B

SOLUTION:

Choice A

If (P, Q, R) = (7, 7, 7), then there will be 7 cubicles having the same co-ordinates x, y and
z viz. (1, 1, 1), (2, 2, 2) ……(7, , 7).

Since, 2 toffees is given to all such cubicles, A = 14.

Similarly, in the second case, x can take only 1 value i.e. 7, y can take 3 values i.e. 6, 7
and 8, while z can take 4 values i.e. 6, 7, 8 and 9.

Hence, the total number of cubicles satisfying the above criteria would be 1 × 3 × 4 = 12.

Since, 1 biscuit is given to all such cubicles, B = 12.

Hence, A > B.

8. If (P, Q, R) = (9, 9, 9) and the number of toffees placed in any cubicle is equal to the
sum of all the co-ordinates of that cubicle. If ―F‖ represents a set of all the possible
values of number of toffees to be kept in cubicles, what is the number of elements in
set F?
A) 729
B) 450
C) 360
D) None of these

Answer Key: D) None of these

SOLUTION:

Choice D

(P, Q, R) cubicle will get number of toffees = P+Q+R

If we observe closely, the minimum summation that we can get is in the cubicle (1,1,1) =
3 and maximum summation will be (9,9,9) = 27

In between (1,1,1) and (9,9,9), all the cubicles will have summation which is between 3
and 27. Hence, total number of combinations possible will be from 3 to 27 = 25
combinations

Directions for questions 9 to 12: In a particular game called Lucky Guy, as many paper
chits are made, as there are players in the game. Some chits have ‗Lucky Guy‘ written on
them, the rest are blank. The chits are folded, such that what is written is not visible, and
then shuffled and distributed, one to each player. All those, who get a chit with ‗Lucky
Guy‘ written on them succeed to the next round and those who get blank chits are
eliminated. Thus at least one player is eliminated in each round. Further, the ‗Lucky Guy‘
chits are reduced in each successive round till at last there is just one winner and all the
other participants are eliminated. While ‗Lucky Guy‘ chits are removed, more blank chits
may be needed so that the number of chits equals the number of players. About a
particular game, we have the following data:

A. The number of ‗Lucky Guy‘ chits removed in each round is a different number. The
number of ‗Lucky Guy‘ chits in the fifth round was three less than in the fourth round.

B. The winner was decided in 5 rounds.

C. To start with, i.e. in round 1, there were 12 ‗Lucky Guy‘ chits.

D. The ratio of the number of participants in round 3 and round 4 is 2 : 1.


9. Type the answer in the given text box:
The number of Lucky Guy chits in round 3 were how many less than in round 2?

Answer Key: 5

SOLUTION:

To decide the winner, in the 5th round there necessarily needs to be only 1 Lucky Guy
chit. Since the game was started with 12 Lucky Guy chits, a total of 11 Lucky Guy chits
have to be removed cumulatively at the end of round I, II, III and IV. Since the number of
Lucky Guy chits removed is different for each round, they have to be 1, 2, 3 and 5 (no
other quartet of 4 different numbers add up to 11).

Since the number of Lucky Guy chits in Round V was 3 less than that in Round IV, the
number of Lucky Guy chits in Round IV is 4 and at end of Round IV, 3 Lucky Guy chits
would have been removed.
The number of participants in any round is equal to the number of Lucky Guy chits in the
previous round) Since the ratio of number of participant in Round III and Round IV is 2 :
1, this will also be the ratio of number of Lucky Guy chits in Round II and Round III.

Checking different possibilities, the number of Lucky Guy chits cannot be 8 & 4 or 12 &
6. If the numbers are 10 & 5, then the number of Lucky Guy chits in successive rounds
will be 12, 10, 5, 4, 1. This means the number of Lucky Guy chits removed at end of
successive rounds is 2, 5, 1, 3. This is exactly in accordance with different number of
chits being removed at end of each round.

Now all the questions can be answered directly.

10. Type the answer in the given text box:


How many participants were eliminated in the last round?

Answer Key: 3

SOLUTION:

To decide the winner, in the 5th round there necessarily needs to be only 1 Lucky Guy
chit. Since the game was started with 12 Lucky Guy chits, a total of 11 Lucky Guy chits
have to be removed cumulatively at the end of round I, II, III and IV. Since the number of
Lucky Guy chits removed is different for each round, they have to be 1, 2, 3 and 5 (no
other quartet of 4 different numbers add up to 11).

Since the number of Lucky Guy chits in Round V was 3 less than that in Round IV, the
number of Lucky Guy chits in Round IV is 4 and at end of Round IV, 3 Lucky Guy chits
would have been removed.
The number of participants in any round is equal to the number of Lucky Guy chits in the
previous round) Since the ratio of number of participant in Round III and Round IV is 2 :
1, this will also be the ratio of number of Lucky Guy chits in Round II and Round III.

Checking different possibilities, the number of Lucky Guy chits cannot be 8 & 4 or 12 &
6. If the numbers are 10 & 5, then the number of Lucky Guy chits in successive
rounds will be 12, 10, 5, 4, 1. This means the number of Lucky Guy chits removed at end
of successive rounds is 2, 5, 1, 3. This is exactly in accordance with different number of
chits being removed at end of each round.

Now all the questions can be answered directly.

11. Type the answer in the given text box:


Find the number of participants in round 4.

Answer Key: 5

SOLUTION:

5
To decide the winner, in the 5th round there necessarily needs to be only 1 Lucky Guy
chit. Since the game was started with 12 Lucky Guy chits, a total of 11 Lucky Guy chits
have to be removed cumulatively at the end of round I, II, III and IV. Since the number of
Lucky Guy chits removed is different for each round, they have to be 1, 2, 3 and 5 (no
other quartet of 4 different numbers add up to 11).

Since the number of Lucky Guy chits in Round V was 3 less than that in Round IV, the
number of Lucky Guy chits in Round IV is 4 and at end of Round IV, 3 Lucky Guy chits
would have been removed.

The number of participants in any round is equal to the number of Lucky Guy chits in the
previous round) Since the ratio of number of participant in Round III and Round IV is 2 :
1, this will also be the ratio of number of Lucky Guy chits in Round II and Round III.

Checking different possibilities, the number of Lucky Guy chits cannot be 8 & 4 or 12 &
6. If the numbers are 10 & 5, then the number of Lucky Guy chits in successive rounds
will be 12, 10, 5, 4, 1. This means the number of Lucky Guy chits removed at end of
successive rounds is 2, 5, 1, 3. This is exactly in accordance with different number of
chits being removed at end of each round.

Now all the questions can be answered directly.

12. For which of the following represents the number of ‗Lucky Guy‘ chits removed
between the rounds in the correct order?
A) 2, 5, 1, 3
B) 1, 5, 3, 2
C) 3, 1, 5, 2
D) 3, 2, 5, 1

Answer Key: A) 2, 5, 1, 3

SOLUTION:

Choice A

To decide the winner, in the 5th round there necessarily needs to be only 1 Lucky Guy
chit. Since the game was started with 12 Lucky Guy chits, a total of 11 Lucky Guy chits
have to be removed cumulatively at the end of round I, II, III and IV. Since the number of
Lucky Guy chits removed is different for each round, they have to be 1, 2, 3 and 5 (no
other quartet of 4 different numbers add up to 11).

Since the number of Lucky Guy chits in Round V was 3 less than that in Round IV, the
number of Lucky Guy chits in Round IV is 4 and at end of Round IV, 3 Lucky Guy chits
would have been removed.
The number of participants in any round is equal to the number of Lucky Guy chits in the
previous round) Since the ratio of number of participant in Round III and Round IV is 2 :
1, this will also be the ratio of number of Lucky Guy chits in Round II and Round III.

Checking different possibilities, the number of Lucky Guy chits cannot be 8 & 4 or 12 &
6. If the numbers are 10 & 5, then the number of Lucky Guy chits in successive rounds
will be 12, 10, 5, 4, 1. This means the number of Lucky Guy chits removed at end of
successive rounds is 2, 5, 1, 3. This is exactly in accordance with different number of
chits being removed at end of each round.

Now all the questions can be answered directly.

Directions for questions 13 to 16: Answer these questions on the basis of the
information given below.

Institute-wise break-up of fees collected by leading B-Schools in the year 2009-10 and
also the break-up of number of students enrolled at these institutes in the year 2010-11
Fees Collected in 2009-10 Students Enrolled in 2010-11

It is also known that the total fees collected in 2010-11 is 2.5 times that in 2009-10

13. If the ratio of the number of students enrolled with institute E in 2009-10 and 2010-
11 is 7 : 8, and the average fees per student collected by the institute in both years is
the same, then by what percentage have its total fees collected by institute E
increased from 2009-10 to 2010-11?
A) 25%
B) 16.67%
C) 14.28%
D) 12.5%

Answer Key: C) 14.28%

SOLUTION:

Choice C

Let the total fees collected by all institutes in 2009-10 be 200 and that in 2010-11 be 500
(i.e. 2.5 times).The fees collected by different institutes in 2009-10 will then be:

Let the number of students with institute E be 7 in 2009-10 and 8 in 2010-11.

Since, the average fees collected by the institute is the same, 287= x8 (x = Fees collected
by E in 2010-11).
x = 32

Hence, the percentage increase in the amount of fees collected by the institute
= (32−28)28 = 14.28%.

14. If the ratio of market share of institute A in terms of fees collected in 2009-10 to that
in 2010-11 is 3 : 5, find the percentage increase in the number of students enrolled in
this institute given that the average fees collected per student by the institute in both
these years is the same.
A) 266.67%
B) 283.5%
C) 307.5%
D) 316.67%

Answer Key: D) 316.67%

SOLUTION:

Choice D

Let the total fees collected by all institutes in 2009-10 be 200 and that in 2010-11 be 500
(i.e. 2.5 times).

The fees collected by different institutes in 2009-10 will then be:


Market Share of A in 2009-10 is 21%. Hence, market share of A in 2010-11 will be 35%
(as the ratio of market share is 3 : 5).

Thus, the actual fees collected by institute A in 2010-11 = 35% of 500 = 175.

Let the number of students with institute A in 2009-10 and 2010-11 be x and y
respectively.

Since, the average fees collected by the institute is the same for both years, 42x=175y

Thus y x=17542= 256

Hence, percentage increase in the number of students = (25−6)6= 196=316.67%

15. If the number of students has remained the same from 2009-10 and 2010-11, and the
institute C has shown an increase of 33.33% in the number of students enrolled by it
and the average fees per student collected by it has only increased by 25%, find the
percentage change in its market share from 2009-10 to 2010-11.
A) 50%
B) 33.33%
C) 25%
D) 20%

Answer Key: B) 33.33%

SOLUTION:

Choice B

Let the total fees collected by all institutes in 2009-10 be 200 and that in 2010-11 be 500
(i.e. 2.5 times).

The fees collected by different institutes in 2009-10 will then be:

Let the total number of students enrolled in both the years be 100.

So, C has 12 students in 2010-11 and 9 students in 2009-10 (since it has increased by
33.33%).

Let the amount of fees collected by C in 2010-11 be ‗x‘.


The average fees per student collected by C in both these years
= 489 and x12 respectively.

But, since the average fees collected per student has increased by
25%, x12=1.25 ×489 or x = 80.

Thus, fees collected by C in 2010-11 is 80 out of 500 i.e. 16%.

Its market share was 24% in 2009-10.

Hence percentage change in its market share = (24−16)24 ×100 = 33.33% decrease

16. If all the institutes collected equal fees in 2010 – 11, the what is the ratio of fees
collected by institute C in 2009 – 10 to the fees collected by institute B in 2010 – 11?
A) 6 : 7
B) 12 : 35
C) 72 : 125
D) None of these

Answer Key: C) 72 : 125

SOLUTION:

Choice C

Let the total fees collected by all institutes in 2009-10 be 200 and that in 2010-11 be 500
(i.e. 2.5 times).The fees collected by different institutes in 2009-10 will then be:
In 2010 – 11, all the institutes collected equal fees of (500 ÷ 6) Hence the required ratio =
48 : 5006=72 :125

Direction for Questions 17 – 20: Each of Arun, Bharat, Chandu and Dharma play one
outdoor sports and one hobby concerned with fine arts. Two play Hockey and two play
Cricket, two are painters, one is a singer and one is a dancer.

Arun is not a painter nor does he play Hockey.

Bharat does not dance.

The dancer plays Hockey.

Bharat and Dharma do not play Cricket.

17. Who among the following plays Cricket and is a painter?


A) Arun
B) Bharat
C) Chandu
D) Arun & Chandu
Answer Key: C) Chandu

SOLUTION:

Choice C

Arun does not play Hockey. So he must play Cricket. Also it is given that he is not a
painter. And since the dancer plays Hockey. Arun cannot be the dancer. Hence he has to
be the singer.

Bharat and Dharma do not play Cricket. Hence they have to play Hockey.
Thus Chandu has to be the other person who plays Cricket.

Bharat does not dance and the dancer plays Hockey. Sodancer has to be Dharma.

Thus, we will have

18. Who among the following is a singer?


A) Dharma
B) Arun
C) Chandu
D) Bharat

Answer Key: B) Arun

SOLUTION:

Choice B
Arun does not play Hockey. So he must play Cricket. Also it is given that he is not a
painter. And since the dancer plays Hockey. Arun cannot be the dancer. Hence he has to
be the singer.

Bharat and Dharma do not play Cricket. Hence they have to play Hockey.
Thus Chandu has to be the other person who plays Cricket.

Bharat does not dance and the dancer plays Hockey. So dancer has to be Dharma.

Thus, we will have

19. Which pair shows the correct relationship of the game and hobby of Dharma?
A) Hockey, Singing
B) Cricket, Dancing
C) Cricket, Painting
D) Hockey, Dancing

Answer Key: D) Hockey, Dancing

SOLUTION:

Choice D
Arun does not play Hockey. So he must play Cricket. Also it is given that he is not a
painter. And since the dancer plays Hockey. Arun cannot be the dancer. Hence he has to
be the singer.

Bharat and Dharma do not play Cricket. Hence they have to play Hockey.
Thus Chandu has to be the other person who plays Cricket.

Bharat does not dance and the dancer plays Hockey. So dancer has to be Dharma.

Thus, we will have

20. Which of the following statement is true about the four friends?
A) Two friends share the same outdoor and fine art interest
B) Those who paint have the same outdoor interest
C) Those who play Hockey have the same interest in fine arts
D) None of the above.

Answer Key: D) None of the above.

SOLUTION:

Choice D
Arun does not play Hockey. So he must play Cricket. Also it is given that he is not a
painter. And since the dancer plays Hockey. Arun cannot be the dancer. Hence he has to
be the singer.

Bharat and Dharma do not play Cricket. Hence they have to play Hockey.
Thus Chandu has to be the other person who plays Cricket.

Bharat does not dance and the dancer plays Hockey. So dancer has to be Dharma.

Thus, we will have

Directions for questions 21 to 24: Answer the following question based on the
information given below.

The following table is a row wise and column arrangement of numbers.

The game has the following rules.


- Your initial value or score is the value of the cell that you start with.

-A higher row or column is one whose subscript is higher than the comparator cell.

-If you move to a higher row, you add the current score with the value of the new cell,
and if you move to a lower row, you subtract the value of the new cell from the current
score.

-If you move to a higher column, you multiply the current value of the cell to the new
cell, and when you move to a lower column, you divide the current value with the value
of the new cell and take the quotient as the new - new value.

-The player can move only one column or one row at a time, and cannot move
diagonally.

21. What is your score if your player moves Row 1 Column 1 ---> One row higher --->
One column higher ---> one column lower?
A) 0
B) 1
C) -1
D) None of these

Answer Key: A) 0

SOLUTION:

Choice A

R1C1---> R2C1---> R2C2---> R2C1

Score (5−3)×0−3=0

22. Type the answer in the given text box:


Player X starts from Column 1 Row 3 and ends up with a negative value in the
minimum possible number of moves. How many moves has X made?
Answer Key: 3

SOLUTION:

The minimum possible number of moves is 3.

R3C1---> R3C2---> R2C2---> R1C2

Score = (2x1) – 0 – 4 = – 2

23. Player Y starts with a negative score and within 1 move, becomes positive. He must
now be on the cell
A) R1C3
B) R2C3
C) R1C2
D) R1C1

Answer Key: B) R 2 C 3

SOLUTION:

Choice B

The player should either have started with R2C1 or R1C3. If he starts with R2C1, he cannot
get positive score within a move. Hence he should have started with R1C3. He must be
now on the cell R2C3.

24. Type the answer in the given text box:


Player Z starts from Row 1 Column 1 and within ‗x‘ moves finds the value of his
player becoming infinity ? What is the minimum possible value of ‗x‘?

Answer Key: 4

SOLUTION:
4

If the score should become infinity, player‘s last move should have been from R2C3 to
R2C2. He has to make 4 moves. R1C1---> R1C2---> R1C3---> R2C3---> R2C2.

Direction for Questions 25 –28: In an Art Exhibition held during Independence Day
celebrations in Delhi, there are five portraits being displayed of well-known freedom
fighters namely Mahatma Gandhi, Bhagat Singh, LalBahadurShastri, LalaLajpatRai and
Jawaharlal Nehru. All these portraits are of different colors: red, orange, blue, violet,
pink. (Not in order) All these five portraits are displayed with different story books, with
each portrait having at least one story book.

Some additional information is given below:

- The number of story books displayed at the violet colored portrait is equal to two-fifth
of the absolute difference between the number of story books displayed at the portraits
of LalaLajpatRai and Jawaharlal Nehru.

- Portraits of LalaLajpatRai and Jawaharlal Nehru are not in violet color.

- The number of story books displayed at the blue colored portrait is one third of the sum
of the number of story books displayed at the portraits ofBhagat Singh
and LalBahadurShastri.

- Portraits of Bhagat Singh and LalBahadurShastri are not in blue color.

- The number of story books displayed at the Jawaharlal Nehru portrait is half the number
of story books displayed at the pink colored portrait.

- Mahatma Gandhi portrait is neither red colored nor violet colored.

- The number of story books displayed at the Jawaharlal Nehru portrait is equal to the
sum of the number of story books displayed at the red and blue colored portraits.

- Jawaharlal Nehru portrait is neither red colored nor blue colored.

Answer the following questions on the basis of the information given above.
25. Type the answer in the given text box:
Color(s) of how many portraits can be determined uniquely?

Answer Key: 1

SOLUTION:

From the additional information given:

I. No. of story books displayed at the violet colored portrait = (25) (no. of story books
displayed at LalaLajpatRai portrait – no. of story books displayed at the Jawaharlal Nehru
portrait)

II. No. of story books displayed at the blue colored portrait) = (13) (Bhagat Singh
+ LalBahadurShastri)

III. No. of story books displayed at Jawaharlal Nehru portrait = (12) (No. of story books
displayed at pink colored portrait)

IV. No. of story books displayed at the Jawaharlal Nehru portrait = (No. of story books
displayed at the red colored portrait) + (No. of story books displayed the blue colored
portrait)

Hence, color of only Jawaharlal Nehru portrait can be determined uniquely.


26. Type the answer in the given text box:
If LalBahadurShastri portrait is Pink in color and the absolute difference between the
number of story books displayed at LalaLajpatRai portrait and Jawaharlal Nehru
portrait is 10, find the number of story books displayed at Mahatma Gandhi portrait.

Answer Key: 10

SOLUTION:

10

From the additional information given:

I. No. of story books displayed at the violet colored portrait = (25) (no. of story books
displayed at LalaLajpatRai portrait – no. of story books displayed at the Jawaharlal Nehru
portrait)

II. No. of story books displayed at the blue colored portrait) = (13) (Bhagat Singh +
LalBahadurShastri)

III. No. of story books displayed at Jawaharlal Nehru portrait = (12) (No. of story books
displayed at pink colored portrait)

IV. No. of story books displayed at the Jawaharlal Nehru portrait = (No. of story books
displayed at the red colored portrait) + (No. of story books displayed the blue colored
portrait)

If LalBahadurShastri portrait is Pink in color⇒


As it is given that | Jawaharlal Nehru – LalaLajpatRai | = 10 and from the question, No.
of story books displayed at the portrait of Jawaharlal Nehru = No. of story books
displayed at the portrait of LalaLajpatRai + No. of story books displayed the portrait of
Mahatma Gandhi.

Hence, No. of story books displayed at the Jawaharlal Nehru portrait - No. of story books
displayed at the LalaLajpatRai portrait = 10 = No. of story books displayed at the
Mahatma Gandhi portrait.

27. Which of the following statement(s) is/are necessarily TRUE?


(i) The violet colored portrait cannot be
of LalaLajpatRaior Jawaharlal Nehruor Mahatma Gandhi.

(ii) The blue colored portrait can be of LalBahadurShastri.

(iii) The red colored portrait cannot be of LalaLajpatRai.


A) Only (i)
B) Only (ii)
C) Only (iii)
D) Both (i) & (iii)

Answer Key: A) Only ( i )

SOLUTION:
Choice A

From the additional information given:

I. No. of story books displayed at the violet colored portrait = (25) (no. of story books
displayed at Lala Lajpat Rai portrait – no. of story books displayed at the Jawaharlal
Nehru portrait)

II. No. of story books displayed at the blue colored portrait) = (13) (Bhagat Singh + Lal
Bahadur Shastri)

III. No. of story books displayed at Jawaharlal Nehru portrait = (12) (No. of story books
displayed at pink colored portrait)

IV. No. of story books displayed at the Jawaharlal Nehru portrait = (No. of story books
displayed at the red colored portrait) + (No. of story books displayed the blue colored
portrait)

Hence, the violet colored portrait cannot be of LalaLajpatRai or Jawaharlal Nehru or


Mahatma Gandhi is only true.

28. How many of the following statements, help you to uniquely determine colour of all
the portraits?

1. LalaLajpatRai portrait is red coloured


2. Portrait of Bhagat Singh is not pink coloured

3. Portrait of Mahatma Gandhi is not orange coloured


A) 1
B) 2
C) 3
D) None

Answer Key: B) 2

SOLUTION:

Choice B

From the additional information given:

I. No. of story books displayed at the violet colored portrait = (25) (no. of story books
displayed at LalaLajpatRai portrait – no. of story books displayed at the Jawaharlal Nehru
portrait)

II. No. of story books displayed at the blue colored portrait) = (13) (Bhagat Singh +
LalBahadurShastri)

III. No. of story books displayed at Jawaharlal Nehru portrait = (12) (No. of story books
displayed at pink colored portrait)

IV. No. of story books displayed at the Jawaharlal Nehru portrait = (No. of story books
displayed at the red colored portrait) + (No. of story books displayed the blue colored
portrait)
If we use First statements, we can conclude colour of LalaLajpatRai as red and Mahatma
Gandhi as blue. With the help of second statement, we can
identify colour of LalBahadurShastri portrait is Pink and colour of Bhagat Singh portrait
is violet. So first two Statements can together help us to uniquely determine colour of all
the portrait. However date of statement (3) is not helpful as we already know portrait of
Jawaharlal Nehru is orange coloured.

Direction for questions 29 - 32: Answer the following questions based on the
information given below.

Mr.Bankatlal acted as a judge for the beauty contest. There were four participants, viz.,
Ms. Andhra Pradesh, Ms. Uttar Pradesh, Ms. West Bengal and Ms.
Maharashtra. Mrs.Bankatlal, who was very anxious about the results, asked him about it
as soon as he was back home. Mr.Bankatlal just said that the one who was wearing the
yellow saree had won the contest. When Mrs.Bankatlal pressed for further details, he
elaborated as follows:

i. All of them were sitting in a row.

ii. All of them wore sarees of different colours, viz., Green, Yellow, White and Red.

iii. There was only one runner-up and she was sitting beside Ms. Maharashtra.

iv. The runner-up was wearing the Green saree.


v. Ms. West Bengal was not sitting at the ends and was not the runner-up.

vi. The winner and the runner-up were not sitting adjacent to each other.

vii. Ms. Maharashtra was wearing a white saree.

viii. Ms. Andhra Pradesh was not wearing the Green saree.

ix. The participants wearing the Yellow saree and the White saree were at the ends.

29. Who wore the Red saree?


A) Ms. Andhra Pradesh
B) Ms. West Bengal
C) Ms. Uttar Pradesh
D) Cannot be determined

Answer Key: B) Ms. West Bengal

SOLUTION:

Choice B

Consider the following statements:

1. All of them were sitting in a row.

2. All of them wore sarees of different colours, viz., Green, Yellow, White and Red.

3. There was only one runner-up and she was sitting beside Ms. Maharashtra.

4. The runner-up was wearing the Green saree.

5. Ms. West Bengal was not sitting at the ends and was not the runner-up.

6. The winner and the runner-up were not sitting adjacent to each other.

7. Ms. Maharashtra was wearing a white saree.


8. Ms. Andhra Pradesh was not wearing the Green saree.

9. The participants wearing the Yellow saree and the White saree were at the ends.

10. The winner was wearing Yellow saree (This is obtained from the facts that the
runner-up was sitting next to Ms. Maharashtra and that the winner was not seated next to
the runner up. Thus, the winner would have to be at the other end. The person at the other
end was wearing a yellow saree).

Using (1), (2), (7), (9) and (10), we have

Using (3) and (4), we have

Using (5) and (8), we have the final arrangement as:


Thus, Ms. West Bengal wore the red saree.

30. Ms. West Bengal was sitting adjacent to …


A) Ms. Andhra Pradesh and Ms. Maharashtra
B) Ms. Uttar Pradesh and Ms. Maharashtra
C) Ms. Andhra Pradesh and Ms. Uttar Pradesh
D) Ms. Uttar Pradesh only

Answer Key: C) Ms. Andhra Pradesh and Ms. Uttar Pradesh

SOLUTION:

Choice C

Consider the following statements:

1. All of them were sitting in a row.

2. All of them wore sarees of different colours, viz., Green, Yellow, White and Red.

3. There was only one runner-up and she was sitting beside Ms. Maharashtra.

4. The runner-up was wearing the Green saree.

5. Ms. West Bengal was not sitting at the ends and was not the runner-up.

6. The winner and the runner-up were not sitting adjacent to each other.

7. Ms. Maharashtra was wearing a white saree.


8. Ms. Andhra Pradesh was not wearing the Green saree.

9. The participants wearing the Yellow saree and the White saree were at the ends.

10. The winner was wearing Yellow saree (This is obtained from the facts that the
runner-up was sitting next to Ms. Maharashtra and that the winner was not seated next to
the runner up. Thus, the winner would have to be at the other end. The person at the other
end was wearing a yellow saree).

Using (1), (2), (7), (9) and (10), we have

Using (3) and (4), we have

Using (5) and (8), we have the final arrangement as:


From the final table obtained in the solution to the first question, Ms. West Bengal was
sitting adjacent to Ms. Uttar Pradesh and Ms. Andhra Pradesh.

31. Which saree was worn by Ms. Andhra Pradesh?


A) Yellow
B) Red
C) Green
D) White

Answer Key: A) Yellow

SOLUTION:

Choice A

Consider the following statements:

1. All of them were sitting in a row.

2. All of them wore sarees of different colours, viz., Green, Yellow, White and Red.

3. There was only one runner-up and she was sitting beside Ms. Maharashtra.

4. The runner-up was wearing the Green saree.


5. Ms. West Bengal was not sitting at the ends and was not the runner-up.

6. The winner and the runner-up were not sitting adjacent to each other.

7. Ms. Maharashtra was wearing a white saree.

8. Ms. Andhra Pradesh was not wearing the Green saree.

9. The participants wearing the Yellow saree and the White saree were at the ends.

10. The winner was wearing Yellow saree (This is obtained from the facts that the
runner-up was sitting next to Ms. Maharashtra and that the winner was not seated next to
the runner up. Thus, the winner would have to be at the other end. The person at the other
end was wearing a yellow saree).

Using (1), (2), (7), (9) and (10), we have

Using (3) and (4), we have

Using (5) and (8), we have the final arrangement as:


From the final table obtained in the solution to the first question, Ms. Andhra Pradesh
wore the Yellow saree.

32. Who was runner – up in the beauty contest?


A) Ms. Andhra Pradesh
B) Ms West Bengal
C) Ms. Utter Pradesh
D) Ms. Maharashtra

Answer Key: C) Ms. Utter Pradesh

SOLUTION:

Choice C

Consider the following statements:

1. All of them were sitting in a row.

2. All of them wore sarees of different colours, viz., Green, Yellow, White and Red.

3. There was only one runner-up and she was sitting beside Ms. Maharashtra.

4. The runner-up was wearing the Green saree.

5. Ms. West Bengal was not sitting at the ends and was not the runner-up.

6. The winner and the runner-up were not sitting adjacent to each other.
7. Ms. Maharashtra was wearing a white saree.

8. Ms. Andhra Pradesh was not wearing the Green saree.

9. The participants wearing the Yellow saree and the White saree were at the ends.

10. The winner was wearing Yellow saree (This is obtained from the facts that the
runner-up was sitting next to Ms. Maharashtra and that the winner was not seated next to
the runner up. Thus, the winner would have to be at the other end. The person at the other
end was wearing a yellow saree).

Using (1), (2), (7), (9) and (10), we have

Using (3) and (4), we have

Using (5) and (8), we have the final arrangement as:


Thus, Ms utter Pradesh was runner – up in the contest
Quantitative Ability

1. If each of the variables x1, x2, x3, x4, ...., x24 can take the values of either +1 or-1, then
what is the minimum non-negative integral value of the sum of the product of the
variables taken two at a time?

Answer Key: 12

SOLUTION:

12

Let n of them be +1 and (24-n) be -1

Product of any two of n variables with value +1 will be +1.

Number of such combinations is

Product of any two of (24 - n)variables with value-1 will be+1

Number of such combinations is

Product of one of (24 - n)variables with value -1 and one of n variables with value +1 is -
1

Number of such combinations is (24 - n) x n

4n2 - 96n+552 =0……(1)

therefore, (n-14.4)*(n-9.6) 0
As n is an integer n = 15 or n 9 for any other value, the value of expression will
only increase.

If n = 15,900 - 1440 + 552 = 12

If n = 9, 324 - 864 + 552 = 12

2. Find the sum up to 30 terms of the sequence, 3, 8, 15, 24 …

Answer Key: 10385

SOLUTION:

10385

if you notice carefully, you can observe that the general term of the series can be written
as n2 + 2nso we need sum of series till t30

This can be found as :

3. Let S={1,2,3,…9}. Find the no. of triplets formed (a, b, c)from the set S such that at
least one of a ,b ,c is 1..?

Answer Key: 217

SOLUTION:

217

No of ways of selecting triplets =9 * 9 * 9


No of ways of selecting triplets with out 1= 8 * 8 * 8

No of ways of satisfying the given condition =729 – 512 = 217

4.
Find the minimum value of

Answer Key: 8

SOLUTION:

(p + q)(q + r)(r + p) 8pqr

Let log a = p

Let log b= q

Let log c = r

log ab = log a + log b = p + q

log bc = log b + log c = q + r

log ca = log c + log a = r + p


5. If P = log35 and Q = log1525, then
A) P > Q
B) P < Q
C) P = Q
D) Q = 2p

Answer Key: A) P > Q

SOLUTION:

The correct choice is A

Q = log1525

Given P = log35

Hence P > 0

so,
P>Q

6. There are some white balls, red balls, black balls and green balls in a box. If green
balls are taken out the number of balls are reduced to half. One third the number of red
balls is equal to one fourth the number of black balls. Thrice the number of red and
black balls is 5 more than four times the number of black balls. Half the number of
white balls is one third the number of green balls. What is the number if balls in the
box?

Answer Key: 42

SOLUTION:

42

Let the number of white , red, black and green, balls be w, g, r & b respectively.

3(3k + 4k) = 4(4k) + 5

k=1 r = 3 and b = 4

3l = r + b + 2l = 7 + 2l l=7

g = 21, w = 14, b = 4 & r = 3 Totally 42 balls

7. N represents a three digit number , where the digits are in arithmetic progression. The
quotient when ―N‖ is divided by the sum of its digits is 48. If 198 is subtracted from N,
the digits are reversed. Find the remainder when ―N‖ is divided by 11.

Answer Key: 3

SOLUTION:
3

Let the digits are (a + d), a ,(a - d).

The sum of digits of ―N‖ is ―3a‖ as the digits are in A.P.

N should be a multiple of 144a (48 x 3a= 144a)

As digits of N reverse when 198 is subtracted, the difference in the hundred‘s place
digit and unit‘s place digit should be 2. Hence d=1

A multiple of 144 which satisfies these condition is 432.

Remainder when 432 is divided by 11 is 3

8. Find the remainder when 2120 is divided by 385.


A) 384
B) 1
C) 330
D) 24

Answer Key: B) 1

SOLUTION:

The correct choice is B

385 = 5 x 7 x 11

24 is divided by 5 leaves a remainder 1

26 is divided by 7 leaves a remainder 1

210 is divided by 11 leaves a remainder 1

2120 is divided by 5 leaves a remainder 1

2120 is divided by 7 leaves a remainder 1


2120 is divided by 11 leaves a remainder 1

2120 is divided by 385 leaves a remainder 1

9. In a parallelogram ABCD, the bisector of angle ABC intersects AD at P. If PD = 5, BP


= 6 and CP = 6, find AB

Answer Key: 4

SOLUTION:

Let AB = y = AP

BC = y + 5

Consider les BAP and BPC

BAP ||| r BPC


y2 + 5y – 36 = 0

y=4

10. In a triangle ABC, B is greater than C. AE is the angular bisector of A. AD is


the perpendicular to BC. If X = (2 x EAD + C) then what is the measure of angle
X?
A) A
B) B
C) AED
D) Cannot be determined

Answer Key: B) B

SOLUTION:

The correct choice is B

Let EAD = and BAD =

EAC = +

AEC = 90 +

From AEC, 90 + + + + c = 1800 or 900 + +2 + C = 1800

From ABD, B + 90 + = 1800 or 90 + = 180 – B


2 + 1800 – B + C = 1800

2a + c = B

11. Three single digit numbers in AP are selected. Using these numbers selected, all
possible 3 digit numbers are formed without repetition. Sum of these numbers is
arrived at. This process is repeated for all selections of single digit numbers in AP.
Let the sum be represented by Si for the ith combination. Any Si will be divisible by

Answer Key: 666

SOLUTION:

666

Sum of all the nos.= (Sum of digits) x (n - 1)! x 111…1

Sum of all the nos.= (a – d + a + a + d) x (3 - 1)! x (111)

Sum of all the nos.= (3a) x (2) x (111) = 666

12.
Given . Find the number of integral solutions to this equation.

Answer Key: 5

SOLUTION:

mn – 97n + 97m = 0

(m – 97) (n + 97) = - 972 = - 9409

First Solution
(m – 97) = - 9409 m = - 9312

(n + 97) = 1 n = - 96

Second Solution

(m - 97) = 1 m = - 96

(n + 97) = - 9409 n = - 9506

Third Solution

(m - 97) = 9409 m = 9506

(n + 97) = - 1 n = -98

Fourth Solution

(m - 97)= - 1 m = 96

(n + 97) = 9409 n = 9312

Fifth Solution

(m - 97) = 97 m = 194

(n + 97) = - 97 n = - 194

Sixth Solution

(m - 97) = - 97 m = 0 not admissible

(n + 97) = 97 n = 0 not admissible

13.
Find the sum of ( infinity)
A) 10.9
B) 11.1
C) 11.9
D) 11.09

Answer Key: A) 10.9

SOLUTION:

The correct choice is A

14. In the adjoining figure, O is the centre and CD is the diameter. CD = 20 units. ABCD
is a quadrilateral and AB = AD = 10 units. Find the area of quadrilateral ABCD.

A) 25
B) 50
C) 25
D) 75

Answer Key: D) 75
SOLUTION:

The correct choice is D

Join OA & OB

OA = OD = OB = OC = 10

AOD = AOB = 600 BOC = 600

Area of the quadrilateral = 3 x 25 = 75

15. A Trapezium circumscribes a circle as shown in the figure below. The non parallel
sides measure 10 units and 6 units. PQ bisects these sides. PQ = . What is the
length of the greatest parallel side if PQ divides the area of the trapezium in the ratio
11:21?

A) 3
B) 8
C) 15
D) 13

Answer Key: D) 13

SOLUTION:
The correct choice is D

Let AB = a and CD = b

Since PQ = , P and Q bisect the lines AD and BC and also the distance between AD
and BC

Area of ABQP =

Area of PQCD =

63a + 21b = 11a + 33b

52a = 12b

AB = 3k, CD = 13k

PQ = 8k

AD + BC = AB + CD

16 = 16k k=1

AB = 3 and CD = 13
16. A and B together can do a piece of work in 15 days. A alone can do the same work in
60 days. A and B work together on this piece of work for 9 days. By that time
the work content reduces by 10% and B's efficiency reduces by 33.33%. In how many
more days will the work get completed?
A) 20/3
B) 6
C) 8
D) 7

Answer Key: B) 6

SOLUTION:

The correct choice is B

Let the work be 60 units

Work = 60 units.

Work done in 9 days = 36 units.

10% of work reduced= 6 units.

Remaining work = 18 units.


Time required = 6 days

17. A circle touches the line x = y, x = -y and point x = 2 + . Find the radius of the
circle.
A) 1unit
B) 2 units
C) units
D) (1 + )units

Answer Key: C) units

SOLUTION:

The correct choice is C

Let ―O‖ be the origin, A be the point of contact of the line x = y with the circle

C be the point (2 + ,0)

OA = OB = r

Let D be the centre of the circle.AD = r

OD = r
OC = r + =r( + 1)

r+r =2+

r=

18. The equation given by [x+2y]2 – 5k[x + 2y]– 14k2 = 0 represents 2 lines. Which of
the following is true?
A) They do not intersect
B) They intersect at 90°
C) They intersect at 60°
D) They intersect at 45°

Answer Key: A) They do not intersect

SOLUTION:

The correct choice is C

[x + 2y]2 – 5k[x + 2y] – 14k2 = 0

Let m = [x + 2y]

m2 – 5km – 14k2 = 0

(m + 2k) (m – 7k) = 0

(m + 2k) = 0 &(m - 7k) = 0 are equations of the 2 straight lines

(x + 2y + 2k) = 0

(x + 2y - 7k) = 0

They are parallel lines and hence do not intersect.

19. There are i cylinders. For any ith cylinder the radius is i and the height is 1 / i. Find
the number of cylinders if the total volume of all cylinders is 990 / 7cm3.
Answer Key: 9

SOLUTION:

Voume of cylinder = r2h

Voume of ith cylinder = = i

Voume of n cylinders =

n(n + 1) = 90

n=9

20. If the perimeter of a regular hexagon is twice that of a circle, what is the ratio of the
area of hexagon to that of the circle?
A)

B)

C)

D)

Answer Key: A)

SOLUTION:

The correct choice is A


Perimeter of the circle =2πr

Perimeter of the Hexagon = 4πr

Side of the Hexagon=

Area of Hexagon=

Area of Hexagon=

Area of the Circle =

21. Akbar, Babar, Shahzahan & Humayun have a total of 148 coins with them. The total
number of coins with Akbar, Babar & Shahzahan is 12 less than that with Babar,
Shahzahan & Humayun combined. If Shahzahan gives as many coins to Babar as the
difference in the number of coins between Akbar & Humayun, then they will
have equal number of coins. What is the total number of coins with Shahzahan &
Humayun together?
Answer Key: 92

SOLUTION:

92

Let Akbar, Babar, Humayun and Shahzahan have a, b, h and s coins respectively.

a + b + s + h = 148…..(1)

a + b + s = (b + s + h) – 12 => a = h – 12 => h – a = 12

Again, s – 12 = b + 12 => s – b = 24 => b = s - 24.

From(1),

h – 12 + s – 24 + s + h = 148

22. How many possible integral solutions does the equation x2 +y2 = x + y + 15 have?
A) Only one
B) Only two
C) Infinitely many
D) None

Answer Key: D) None

SOLUTION:

The correct choice is D

To get solutions which are integers, let the LHS be even, Then either both x and y
are even or both are odd. However in that case the RHS becomes odd which is a
contradiction. If on the other hand, the LHS be odd, then either x or y is odd but not both
can be of the same parity. However in this case the RHS becomes even. Hence no
solutions in integers are possible. Hence the answer is option (D).
23. A person typed all the natural numbers up to 105. He then deleted all the multiples of
3, 5 or 7. How many numbers would finally be visible in the document?

Answer Key: 48

SOLUTION:

48

3, 5 and 7 are the only prime factors of 105, so, if we delete all numbers divisible by any
of them, then the numbers are co-prime to 105 and obviously less than 105. So, basically
we need to find out φ (105) {where φ(x) gives all coprimes of x that are lesser than x}.
So φ(105)= 105 x (1 - 1/3) x (1 – 1/5) x (1 – 1/7) = 105 x (2/3) x (4/5) x (6/7) = 48.

24. What are the last two digits of the number 5151?

Answer Key: 51

SOLUTION:

51

5151 = (50 + 1)51

Last term = 151 = 1

2nd last term = 51C50 x 50 = 51 x 50= 2550

3rd terms onwards all the terms are multiples of 2nd or higher power of 50. So, they will
contribute nothing but 0‘s in last two places. So, last two digits are 51.

25. If a, b and c are real numbers and 8a2 + 37b2 + 18c2 – 20ab - 42bc = 0. Find the

ratio
A)

B)
C)
D) Cannot be determined

Answer Key: C)

SOLUTION:

The correct choice is C

We have,

8a2 + 37b2 + 18c2- 20ab-42bc=0

Multiplying both sides by 2,

16a2+ 74b2 +36 c2 -40ab- 84bc =0

Or 16a2 – 40ab + 25b2 + 49b2 – 84bc +36c2 = 0

Or (4a-5b)2 + (7b-6c)2= 0

this means 4a-5b = 0 and 7b-6c = 0

Since a, b, c are real numbers,

4a-5b=0 or 4a= 5b

When

And we also have

7b-6c =0 or 7b =6c

From (1) and (2), we have,


a : b : c = 15 : 12 : 14

Hence the correct answer is option (C)

26. The speeds of Amar and Akbar are in the ratio 2:1. They are simultaneously moving
up an escaslator, which is also moving up, from ground to first floor. Amar takes 60
steps to reach the top, while Akbar takes 40 steps to reach the top. If the escalator was
static, how many steps would Amar have to take to reach the top?

Answer Key: 120

SOLUTION:

120

Let Amar reach the top in ―t‖ seconds, taking 60 steps. In the same time, Akbar takes 30
steps, as their speed ratio is 2:1. Let the escalator in the same time take X steps.

Hence, w.r.t. Amar, 60 + X = number of steps of the static escalator----------(1)

Now, Akbar needs to take 10 more steps to reach the top, which he will do in another
―t/3‖ seconds, and in the same ―t/3‖ seconds, the escalator will move another X/3 steps.

Hence, w.r.t. Akbar, (30 + X) + (10 + X/3) = number of steps of the static escalator-------
(2)

Solving, we get X = 60.

Hence, number of steps of the static escalator = 120

27. Harsh, Madhu and Anshu can complete a piece of work in 90, 60 and 45 days
respectively. After working for ―x‖ days together, Harsh left. After ―y‖ more days
Madhu left and Anshu completes the remaining work. Had Madhu not left they could
have completed the work in (y + 4) days after Harsh had left. Had none of them left,
then they would have completed the work in x+7 days. Find the total number of days
required to complete the work.
Answer Key: 25

SOLUTION:

25

Let the total work be 180 units (LCM of 90,60,45)

Therefore efficiency of Harsh, Madhu and Anshu are 2units/day , 3units/day and 4
units/day respectively.

If none of them leave, the total work can get completed in 180 /(2+3+4) = 20 days

According to the question x+7 = 20 days. Therefore x =13.

So they worked together for 13 days in which they complete 13 x (2+3+4) = 117 units.

Remaining units = 180 – 117 = 63.

If Madhu doesn‘t leave, Anshu and Madhu can complete the remaining work in 63 /
(3+4) = 9 days.

According to the given question y+4 = 9 , therefore y = 5 days.

In 5 days, Madhu and Anshu complete 7 x 5 = 35 units.

Therefore amount left = 63 -35 = 28 units.

This 28 units is completed by Madhu in 28/4 = 7 days.

Therefore the total time to complete the work = (x + y +7 ) = (13 +5 +7) =25 days

28. The extreme value of a quadratic expression occurs at x = 6. One of the roots occurs
at the point x = 2. Find the other root.
A) 2
B) 10
C) 8
D) Cannot be determined
Answer Key: B) 10

SOLUTION:

The correct choice is B

If the graph of any quadratic expression is plotted, the real roots (if any) lie
symmetrically about the point of extreme. Hence we have, if the other root is y,

then,

Hence y = 10. Hence the answer is option (B)

29. In how many ways can u place six identical Indian 5 rupee coins on the six vertices of
a regular hexagon?
A) 1;
B) 26 ;
C) 5!;
D) None of these

Answer Key: D) None of these

SOLUTION:

The correct choice is D

Since the coins are identical and the hexagon is regular, it might seem that the answer to
this question is 1.

But it isn‘t. It would have been one if the two sides of the coins are same. But Indian 5
rupee coin has a head and a tail. So, depending on which side is facing up, we will get
different arrangements.

We need to consider these cases –

1. 6 Heads and 0 Tails - 1 way

2. 5Heads and 1 Tail - 1 way


3. 4 Heads and 2 Tails - 3 ways [The two tails can be adjacent to each other or there
might be a head between those two tails; or 2 heads between 2 tails]

4. 3 Heads and 3 Tails – 3 ways [The arrangements are shown in the diagram below]

5. 2 Heads and 4 Tails – 3 ways

6. 1 Head and 5 Tails – 1 way

7. 0 Head and 6 Tails – 1 way

7Hence the total arrangements that are possible are (1+1+3+3+3+1+1) = 13. Hence
option (d).

30. A polynomial leaves a remainder of 5 when divided by x - 3 and a remainder of -7


when divided by x + 1. What is the remainder when the polynomial is divided by x2 -
2x - 3?
A) 3x – 4;
B) 3x + 4;
C) 4x - 3 ;
D) None of the above

Answer Key: A) 3x – 4;
SOLUTION:

The correct choice is A

We observe that when we divide by a second degree polynomial the remainder will
always be linear or constant.

Thus the division statement becomes

p(x) = (x2 - 2x - 3)q(x) + ax + b......................(eqn.1)

where p(x) is the polynomial, q(x) is the quotient polynomial and ax + b is the remainder.
Now we observe that the remainder theorem states p(3) = 5 and p(-1) = -7. Also we
notice that x2 - 2x - 3 = (x - 3)(x + 1).

Thus substituting x = 3 and -1 into (eqn.1) we have:

p(3) = 5 = 3a + b

p(-1) = -7 = -a + b

Solving these equations a = 3 and b = -4; the remainder is 3x - 4.

31. S is the point on the side QR of a triangle PQR such that angle PSR equal to angle
QPR. The length of the side PR is 15 cm. Find the maximum possible length of QS if
it is known that both QR and RS take integral ( in cm ) values greater than one.
A) 78
B) 75
C) 73
D) 72

Answer Key: D) 72

SOLUTION:

The correct choice is D

The diagram can be drawn as below.


Considering triangles PSR and QPR,

As angle PSR = angle QPR

And angle R is common to both triangles,

Then angle Q would be equal to angle SPR.

Hence the triangles are similar.

Now PR/QR = SR/PR

PR2 = QR . SR

225 need to be written as product of two integers (none of them 1)

225 = 3 x 75

Hence minimum possible value of RS = 3 when QR is 75

Hence Max QS = 72 cm

Option D

32. A sphere is cut into eight equal parts by three mutually perpendicular planes passing
through the centre. If the sphere has unit radius, then calculate the radius of the
largest sphere that can be inscribed in one of the eight parts so created.
A)
B)

C)
D)

Answer Key: A)

SOLUTION:

The correct choice is A

Consider the following figures

The plane AQP in figure 1 is redrawn in figure 2 as a planar section. The plane APQ is so
drawn that, it passes through the centre of the inscribed sphere , namely the point O.Q is
the centre of the sphere that was originally cut into eight equal parts. The plane APQ
forms the sector of a circle of unit radius in which the circle with centre O is inscribed.
The circle with centre O is also the section of the sphere with centre O which is cut by the
plane AQP , hence , its radius is also the required radius. From O we drop a perpendicular
OM on AQ. Let OL= r . Hence OM= r. Now angle PQA= 90o. Hence angle MQO=45o.
Hence .Hence , . Hence as given by
option (A).

33. Consider the following figure,


The above figure is that of an U shaped block of stone such that IJ = 20 units, ID =
JG = 15 units and AB = BC = CD = AD = 5 units. The cross section of the block is
square in shape and the legs of the block are perpendicular to the base i.e. , angle DIJ
= 90o and angle GJI = 90o. Find the length of the shortest possible path joining A and
H along the surface of the block.
A)
B)
C)

D)

Answer Key: B)

SOLUTION:

The correct choice is B

Consider the figures given below


If the given block in figure 1 is cut by a planes through LK and MN parallel to the planes
ABCD and EFGH and if the cut portions are joined to form the cuboid in figure 2, then
DG = 40 units and ABCD would be a square having side 5 units. Hence the shortest paths
can be obtained either join A1H (as in figure 3) or A1H as in figure 4, where A1BCD and

A1CHF1 lie on the same plane. Thus A1H = or


A1 H of which is the shortest as obtained from figure 4.
Hence the answer is option (B).

34. ABC is a right angled triangle with AB the hypotenuse and angle CAB = 25o. CO is a
median, O being the midpoint of AB. Consider a point D , such that BE is bisected by
CO at D and BE and CO are mutually perpendicular. Calculate angle CEB.
A) 15o
B) 20o
C) 250
D) Cannot be determined

Answer Key: C) 25
0
SOLUTION:

The correct choice is C

Consider the following figure. We join CE and AE.

We have ABC is a right angled triangle with angle ACB=90 degrees and CO the median
on side AB. By question D is the midpoint of BE and O the midpoint of AB and CD is
perpendicular to BE. Hence, angle BEA is 90o as OD is parallel to AE. This implies that
triangle EBA is a right angled triangle, right angled at E and AB its hypotenuse. Hence
we can draw a circle with diameter AB and the circle also passes through the points C
and E. Thus angles BAC and BEC are subtended from the same minor arc BC and hence
equal. Hence angle BEC=250 as given in option (C).

Das könnte Ihnen auch gefallen